SlideShare a Scribd company logo
1 of 39
Dermatopathology
1) Both mesenchymal and epithelial elements may be found in this tumor:

A. Bednar tumor

B. Cylindroma

C. Chondroid syringomaCorrect Choice

D. Microcystic adnexal carcinoma

E. Folliculosebaceous cystic hamartoma
Cutaneous mixed tumor, also known as Chondroid syringoma, represents an acquired hamartoma
with folliculosebaceous-apocrine differentiation that has been generally interpreted as a form of
adnexal adenoma (neoplasm). It has both a mesenchymal and epithelial component


2) What is the diagnosis?

A. Papillary eccrine adenoma

B. SyringomaCorrect Choice

C. Adenoid cystic carcinoma

D. Trichoadenoma

E. Dermal duct tumor
This is a syringoma, in which there are small cords and strands of epithelial cells, some in a
“tadpole” configuration. There are scattered lumens, often lined with clear cells, with a bluish
substance within them. No horn cysts are present


3) The diagnosis is:
Image   Link   1
Image   Link   2
Image   Link   3
Image   Link   4


A. clear cell Bowen's disease

B. extramammary Paget'sCorrect Choice

C. epidermotropic sebaceous carcinoma

D. epidermotropic Merkel cell carcinoma

E. epidermotropic balloon-cell melanoma
In extramammary Paget’s disease, there are epidermotropic, large cells with a bluish cytoplasm. At
times these cells can form “nests”, but in general, there is usually a compressed basal layer beneath
the “nests”.


4) This is a desmoplakin:

A. Plakophilin

B. Plakoglobin

C. BPAg1 Correct Choice

D. Beta-catenin




                                                                                                    1
E. Desmocollin
Desmoplakins include desmoplakin 1, BPAg1, envoplakin, and periplakin


5) Histologically, adenoma sebaceum represent which of the following lesions?

A. Angiokeratomas

B. Angiofibromas Correct Choice

C. Collagenomas

D. Neurofibromas

E. Smooth muscle hamartomas
Adenoma sebaceum, fibrous papules and pearly penile papules all have similar features
histologically, presenting as angiofibromas. Features include atrophic epidermis with patchy
melanocytic hyperplasia and hyperkeratosis, vertically oriented collagen, increased fibroblasts and
blood vessels


6) Lipomembranous change is seen in:

A. Sclerosing panniculitis Correct Choice

B. Sebaceous carcinoma

C. Cystic sebaceous adenoma

D. Hibernoma

E. Mucocele
Lipomembranous change is a non-specific histologic pattern that is most commonly seen in
lipodermatosclerosis, which is also known as sclerosing panniculitis; this condition may be
secondary to venous stasis


7) The diagnosis is:
Image Link 1
Image Link 2


A. sarcoidosis

B. foreign body

C. lupus miliaris et disseminata

D. lichen nitidusCorrect Choice

E. lymphocytoma cutis


8) What is the diagnosis?

A. Pilomatrixoma

B. Metastatic carcinoma

C. Tattoo

D. Wood splinter

E. Calcinosis cutisCorrect Choice
This is calcinosis cutis, in which there is calcification (blue chunky material) in the dermis



                                                                                                      2
9) The diagnosis is:

A. Neurilemmoma

B. Granular cell tumor

C. Xanthoma Correct Choice

D. Reticulohistiocytoma

E. Neuroma
NEEDS EXPLANATIONS


10) The diagnosis is:
Image   Link    1
Image   Link    2
Image   Link    3
Image   Link    4


A. spiradenoma

B. mastocytosis

C. hidradenoma

D. glomangiomaCorrect Choice

E. Kimura's
In glomangiomas, one sees multiple lumina lined by cells with pink cytoplasm and indisctinct
borders with very round nuclei. Generally, the lumina are lined by one or two layers of glomus cells.


11) 62-year old female with history of acute myeloid leukemia presents with multiple edematous,
erythematous papules after starting G-CSF.


A. Bowel bypass dermatosis

B. Leukocytoclastic vasculitis

C. Erythema multiforme

D. Polymorphous light eruption

E. Sweet's syndromeCorrect Choice
Sweet's syndrome, or acute febrile neutrophilic dermatoses, is often associated with AML and G-
CSF. Histologically, there is marked dermal edema with a prominent infiltrate composed of
neutrophils with leukocytoclasia. There is an absence of extensive vascular damage


12) Multiple such lesions are seen in this syndrome:
Image Link 1
Image Link 2
Image Link 3


A. Nicolau and Balus'

B. Alagille's

C. Rubinstein-Taybi's

D. Schopf's




                                                                                                    3
E. Brooke-SpieglerCorrect Choice
In Brooke-Spiegler syndrome, there are multiple trichoepitheliomas, cylindromas, and
spiradenomas. Nicolau and Balus’ syndrome has multiple eruptive syringomas, milia, and
atrophoderma vermiculata. Alagille’s syndrome is the association of arteriohepatic dsyplasia with
nevus comdonicus. Schopf’s syndrome associates multiple hidrocystomas with hypodontia,
palmoplantar hyperkeratosis, and onychodystrophy. In Rubinstein-Taybi’s syndrome, patients are
short of stature with broad thumbs and multiple pilomatricomas


13) This patient has multiple lesions showing the following histology. She has a family history of
such lesions. You check her for:
Image Link 1
Image Link 2
Image Link 3


A. fumarate hydrataseCorrect Choice

B. transglutaminase

C. epoxide hydrolase

D. sulfatase

E. urease


14) The diagnosis is::
Image Link 1
Image Link 2
Image Link 3


A. incontinentia pigmentiCorrect Choice

B. pemphigus vulgaris

C. epidermal nevus

D. bullous pemphigoid

E. dermatitis herpetiformis


15) The diagnosis is:
Image Link 1
Image Link 2
Image Link 3


A. adenoid cystic carcinomaCorrect Choice

B. trichoepithelioma

C. dermal duct tumor

D. syringoma

E. tubular apocrine adenoma
In adenoid cystic carcinoma, there are cords and tubules of basaloid cells, arranged in a cribiform
pattern. The cystic spaces contain a bluish material. Perineural invasion is common. The cells stain
positively for epithelial membrane antigen (EMA).


15) The diagnosis is:
Image Link 1




                                                                                                       4
Image Link 2
Image Link 3


A. adenoid cystic carcinomaCorrect Choice

B. trichoepithelioma

C. dermal duct tumor

D. syringoma

E. tubular apocrine adenoma
In adenoid cystic carcinoma, there are cords and tubules of basaloid cells, arranged in a cribiform
pattern. The cystic spaces contain a bluish material. Perineural invasion is common. The cells stain
positively for epithelial membrane antigen (EMA


16) For this patient, you request that the lab perform indirect immunofluorescence using what
substrate?


A. Monkey esophagus

B. Mouse epithelium

C. Rat bladder

D. Guinea pig esophagusCorrect Choice

E. Hep-2 cells
This patient has pemphigus folicaceus, and indirect immunofluorescence works best on guinea pig
esophagus


17) This patient may need blood tests to check her:

A. Renal function

B. White blood cell count

C. Liver function

D. ThyroidCorrect Choice

E. Glucose
Herpes gestationis is associated with an increased incidence of Graves’ disease.


18) The diagnosis is:

A. Wegener’s granulomatosis

B. Syphilis

C. Fixed drug reaction

D. Granuloma faciale Correct Choice

E. Angiolymphoid hyperplasia with eosinophilia
NEEDS EXPLANATIONS


19) Multiple clear cell acanthomas are associated with:




                                                                                                       5
A. Immunosuppression

B. Ichthyosis Correct Choice

C. Gastrointestinal polyps

D. Breast cancer

E. Cowden’s
Clear cell acanthoma is associated with ichthyosis


20) The diagnosis is:
Image Link 1
Image Link 2
Image Link 3


A. metastatic carcinoma

B. Merkel cell carcinoma

C. pyogenic granuloma

D. bacillary angiomatous

E. glomus tumorCorrect Choice
In a glomus tumor, generally vascular spaces are not particularly prominent, and there are cords as
well as solid areas of uniform cells with very monomorphous rounded nuclei


21) The diagnosis is:
Image   Link   1
Image   Link   2
Image   Link   3
Image   Link   4


A. Large cell acanthoma

B. Clear cell acanthomaCorrect Choice

C. Hidroacanthoma simplex

D. Poroma

E. White sponge nevus
In a clear cell acanthoma, there is regular acanthosis, often with some parakeratotic scale with
neutrophils overlying it. The keratinocytes making up the thickened epidermis are often clear, or
pale, due to an increased glycogen content secondary to a deficiency of phosphorylase. There is
often a very abrupt demarcation at the margins of the acanthotic pale/clear cells and the normal
epidermis


22) The gene defect:
Image Link 1
Image Link 2
Image Link 3


A. DSG3

B. ATP2A2

C. ATP2C1Correct Choice

D. SERCA2




                                                                                                    6
E. CHRNA9
In Hailey-Hailey disease, there is full-thickness acantholysis of the epidermis. The gene defect is in
ATP2C1


23) This tumor is vimentin+ and cytokeratin+:

A. Neurothekeoma

B. Dermatofibroma

C. Angiosarcoma

D. Epithelioid sarcoma Correct Choice

E. Nodular fasciitis
Characteristic immunohistochemistry of epithelioid sarcoma is vimentin- and cytokeratin- positivity


24) The diagnosis is:
Image Link 1
Image Link 2
Image Link 3


A. argyriaCorrect Choice

B. minocycline-induced pigmentation

C. hemochromatosis

D. ochronosis

E. gold effect
In argyria, the silver deposits (black particles) are often seen around eccrine glands.


25) The diagnosis is:
Image Link 1
Image Link 2
Image Link 3


A. traumatic neuroma

B. acral angiofibroma

C. accessory digit

D. Koenen's tumor

E. acquired digital fibrokeratomaCorrect Choice
This is an acquired digital fibrokeratoma, with a thickened stratum corneum indicating an acral
location and an absence of nerves in the dermis. Collagen is increased in the dermis


26) With immunofluorescence, the most likely pattern would be:
Image Link 1
Image Link 2
Image Link 3


A. granular IgG

B. granular IgACorrect Choice




                                                                                                         7
C. linear IgG, IgM, C3, and IgA

D. tubular IgG

E. linear IgM
In dermatitis herpetiformis, there are characteristic clusters of neutrophils at the tips of dermal
papillae. Immunofluorescence will show granular IgA in the dermal papillae. Neutrophils at the
dermo-epidermal junction of a bulla can also be seen in bullous systemic lupus erythematosus,
linear IgA disease, neutrophilic bullous pemphigoid, and inflammatory epidermolysis bullosa
acquisita


27) The diagnosis is:
Image Link 1
Image Link 2
Image Link 3


A. epidermal nevus

B. poromaCorrect Choice

C. bowenoid papulosis

D. large cell acanthoma

E. verruca
In a poroma, there is a down-growing epidermal proliferation composed of uniform cells, often with
indistinct cytoplasmic borders that have rounded, monomorphous nuclei. Within this proliferation,
there are often ducts lined by an eosinophilic cuticle


28) Which immunohistochemical stain would be positive in eosinophilic granuloma?

A. Congo red

B. Mucin

C. Cytokeratin 20

D. CD1aCorrect Choice

E. HMB-45
Eosinophilic granuloma is a form of Langerhans Cell Histocytosis (LCH), previously called
Histiocytosis X. Eosinophilic granuloma is a localized, benign form which is more common in males
and generally affects the bones. All forms of LCH are characterized by the infiltration of Langerhans
cells on pathology, which staing for S-100, CD1a and contain cytoplasmic birbeck granules


29) What is the diagnosis?

A. Lichen nitidusCorrect Choice

B. Lupus

C. Lichen planus

D. Lichen planus-like keratosis

E. Lichen striatus
The histologic features shown are indicative of lichen nitidus. Lichen nitidus has a very characteristic
"claw clutching ball" appearance where the rete ridges extend downward around a lichenoid
infiltrate




                                                                                                      8
30) An increased number of miniaturized hairs are seen in:

A. Lichen planopilaris and alopecia areata

B. Alopecia areata Correct Choice

C. Trichotillomania

D. Lichen planopilaris

E. Telogen effluvium
In alopecia areata, especially early stages, increased miniaturized catagen hairs can be seen in
addition to the finding of peribulbar lymphocytes resembling a “swarm of bees.” Lichen planopilaris
is a scarring alopecia in which vertical tracts of fibrosis are seen in place of follicles. Trichotillomania
displays follicular plugging, trichomalacia, pigmented casts, hemorrhage, and increased catagen
hairs on biopsy. Telogen effluvium is characterized by an increased number of telogen hairs


31) The diagnosis is:
Image Link 1
Image Link 2
Image Link 3


A. Monsel's reaction

B. chrysiasis

C. amalgam tattoo

D. minocycline-induced pigmentation

E. ochronosisCorrect Choice
In ochronosis, there is slightly thickened banana-shaped collagen in the superficial dermis that
appears yellow-brown in color


32) The diagnosis of this biopsy is:

A. Erythema nodosum

B. Polyarteritis nodosaCorrect Choice

C. Granuloma annulare

D. Leukocytoclstic vasculitis

E. Reumatoid nodule
Polyarteritis nodosa is a necrotizing vasculitis of medium-sized arteries in the derma-subcutaneous
junction. It is a septal panniculitis as a result of vasculitis. There is more fibrinous changes than in
thrombophlebitis and less necrosis than erythema induratum


33) What is the pathologic diagnosis of this lesion?

A. Acanthosis nigricans

B. Verruca planaCorrect Choice

C. Seborrheic keratosis

D. Normal skin

E. Epidermal nevus




                                                                                                           9
The histologic characteristics of verruca plana are presence of hyperkeratosis and acanthosis.
Koilocytes appear in the upper one-third of the epidermis


34) Similar follicles can be seen on skin from the:
Image Link 1
Image Link 2


A. fiinger

B. glans penis

C. nose

D. labia majora

E. eyelidCorrect Choice
Vellus hair follicles are seen commonly in accessory tragic and eyelid skin


35) The diagnosis is:
Image   Link   1
Image   Link   2
Image   Link   3
Image   Link   4


A. Bowen's disease

B. melanoma

C. sebaceous carcinoma

D. pagetoid reticulosis

E. Paget's diseaseCorrect Choice
In Paget’s disease, there are epidermotropic cells in a variably acanthotic epidermis. The
epidermotropic cells have abundant cytoplasm that is often blue-gray in color. The cells are single
or in nests throughout the epidermis. There should be a compressed rim of basal cells beneath
nests that are located near the dermoepidermal junction


36) The diagnosis is::
Image Link 1
Image Link 2
Image Link 3


A. mixed tumor

B. follicular mucinosisCorrect Choice

C. alopecia areata

D. lichen planopilaris

E. pityrosporum folliculitis


37) Eosinophils are typically found in each of the following except:

A. Incontinentia pigmenti

B. PlasmacytomaCorrect Choice

C. Urticaria



                                                                                                  10
D. Lichenoid drug reactions

E. Pemphigus vulgaris
Plasmacytomas are characterized by the presence of monoclonal plasma cells. Plasmacytomas may
occur from primary cutaneous focus or secondarily from myeloma


38) The diagnosis is:
Image   Link   1
Image   Link   2
Image   Link   3
Image   Link   4
Image   Link   5


A. Merkel cell carcinomaCorrect Choice

B. neuroblastoma

C. plasmacytoma

D. Lymphoma

E. melanoma
This is a Merkel cell carcinoma, in which there is a dense collection of small blue cells with scant
cytoplasm in the dermis. The cells are sometimes arranged in trabeculae and other times in
nodules. The cells appear very blue on low power, and on higher power have somewhat pale nuclei
that have paler/darker foci within them in a salt-and-pepper pattern


39) Which of the following lesions demonstrates a pseudo-Darier’s sign?

A. Mastocytoma

B. Spitz nevus

C. Smooth muscle hamartomaCorrect Choice

D. Pilomatricoma

E. Bullous pemphigoid
Smooth muscle hamartomas are benign tumors which arise from smooth muscle of the dermis.
Pseudo-Darier's sign may be elicited due to transient piloerection after rubbing. Histologically, red-
orange bundles and fascicles are present with blunt-ended nuclei


40) The diagnosis is:
Image Link 1
Image Link 2
Image Link 3


A. apocrine adenoma

B. amyloidosis

C. Masson's tumorCorrect Choice

D. intravascular pyogenic granuloma

E. papillary digital adenocarcinoma


41) The most likely diagnosis for this painful neoplasm is:



                                                                                                    11
A. Glomus tumor

B. Neurilemmoma

C. Cutaneous endometriosisCorrect Choice

D. Blue rubber bleb nevus

E. Angiolipoma
Cutaneous endometriosis usually occurs after gynecologic surgery. The rich, cellular stroma with
regularly shaped channels and glandular structures are typical. Hemorrhage may also be present in
deeper sections


42) What infectious agent is most likely responsible for this reaction of fibrin and antibodies which
help to prevent phagocytosis?


A. ActinomycosisCorrect Choice

B. Anthrax

C. Candida albicans

D. Ricketsii species

E. Nocardia
Hoeppli-Splendore reaction is characterized histologically by intensely eosinophilic material
consisting of fibrin and antibodies. Causes of the phenomenon include Actinomycosis israelii, Staph
aureus, Proteus, Pseudomonas and E. coli


43) Goblet cells are seen in:

A. Bronchogenic cyst Correct Choice

B. Steatocystoma

C. Cutaneous ciliated cyst

D. Dermoid cyst

E. Endometriosis
Bronchogenic cysts have a pseudostratified cuboidal or columnar lining that is ciliated; goblet cells
are found in the lining as well


44) The diagnosis is:

A. Metastatic carcinoma

B. Mixed tumor Correct Choice

C. Papillary eccrine adenoma

D. Nodular hidradenoma

E. Mucinous carcinoma
NEEDS EXPLANATIONS
45) The most common cause of this in the U.S. is:
Image Link 1
Image Link 2
Image Link 3




                                                                                                    12
A. Microsporum distortum

B. Epidermophyton floccosum

C. Trichophyton rubrumCorrect Choice

D. Microsporum canis

E. Trichophyton mentagrophytes
Trichophyton rubrum is the most common cause of Majocchi’s granuloma, a type of folliculitis where
the dermatophyte likely tracks down the follicle and creates a foreign-body-type inflammatory
reaction in the dermis


46) This patient recently developed this rash. You decide to patch test her, but in the meanwhile
you tell her to avoid:


A. Primin

B. Abietic acid

C. Benzocaine

D. CinnamonCorrect Choice

E. Chamomile
This patient likely has a fragrance allergy. Patients allergic to fragrances also need to avoid certain
spices like cinnamon


47) Multiple such lesions may be seen associated with:
Image Link 1
Image Link 2
Image Link 3


A. Cowden's

B. Wermer'sCorrect Choice

C. Cowper's

D. Werner's

E. Brooke's
MEN type I (Wermer's Syndrome) is sometmes associated with multiple angiofibromas. Tuberous
sclerosis is also associated with adenoma sebaceum (angiofibromas


48) Turk cells are found in what infection?

A. Rubeola

B. Mumps

C. Roseola

D. Syphilis

E. RubellaCorrect Choice
Turk cells are atypical lymphocytes found in rubella


49) The diagnosis is:
Image Link 1




                                                                                                      13
Image Link 2
Image Link 3


A. Masson's

B. bacillary angiomatosis

C. hemangioendothelioma

D. fibrosing pyogenic granulomaCorrect Choice

E. apocrine adenoma


50) Which disease process best describes Texier's disease?

A. Deposition disorder

B. PanniculitisCorrect Choice

C. Granulomatous disease

D. Neutrophilic dermatosis

E. Infectious process
Texier's disease is a panniculitis secondary to vitamin K injections causing sclerotic lesions with lilac
borders on the buttocks and thighs resembling a cowboy belt and holster


51) A patient with dystrophic nails and multiple lesions with this histologic finding may have what
gene defect?


A. Phosphorylase

B. Keratin 6b/17Correct Choice

C. Beta-catenin

D. Keratin 6a/16

E. Keratin 1/10
Pachonychia congenita type II (also known as Jackson-Lawler) is a autosomal dominant disorder
characterized by natal teeth, steatocystoma multiplex, and pincer nails. Steatocystomas have cyst
walls that are intricately folded or crenulated. Sebaceous glands are present within the cyst wall


52) What is the diagnosis?

A. Warty dyskeratoma

B. Molluscum contagiosumCorrect Choice

C. Trichilemmoma

D. Ecthyma contagiosum

E. Orf
This is a lesion of Molluscum contagiosum, in which there is a lobulated down-growth (cup-shaped)
of keratinocytes; centrally towards the surface, the keratinocytes are enlarged with cytoplasmic
pink inclusions (Henderson-Patterson bodies


53) Multiple such lesions can be associated with:
Image Link 1




                                                                                                      14
Image Link 2
Image Link 3


A. Gaucher's

B. Sipple's

C. fucosidosisCorrect Choice

D. Louis-Barr

E. Bourneville's
multiple angiokeratomas in a bathing trunk distribution may be associated with Fabry's disease
(angiokeratoma corporis diffusum) or fucosidosis as well as some other storage diseases as well as
possibly being a normal finding. There has been a recent report of angiokeratomas in a bathing
trunk distribution in a woman with no other signs of a storage disorder


54) What is the diagnosis?

A. Lipoid protinosis

B. Erythrpoietic protoporphyria

C. Lichen sclerosus et atrophicusCorrect Choice

D. Lichen amyolidosus

E. Morphea
This is lichen sclerosus et atrophicus in which there is hyperkeratosis overlying an atrophic
epidermis. Underlying the epidermis is a layer of homogenized light pink collagen. Beneath that,
there is a somewhat band-like inflammatory infiltrate of predominantly lymphocytes, but also
histiocytes and plasma cells


55) What is the diagnosis?

A. Seborrheic keratosis

B. Hidroacanthoma simplex

C. Epidermal nevus

D. Fibroepithelioma of PinkusCorrect Choice

E. Nevus sebaceus
Fibroepithelioma of Pinkus is a form of basal cell carcinoma. Histology shows long, thin,
anastomosing strans of basal cell embedded in fibrous stroma with many connections to the
epidermis


56) A newborn infant presents with bullous lesions. Based upon the pathology, what is the most
likely gene defect causing her skin condition?


A. Keratin 1 and 10

B. PAX3

C. NEMOCorrect Choice

D. Keratin 5 and 14

E. SPINK5




                                                                                                   15
Incontinentia pigmenti is an X_linked dominant genodermatosis which presents in the newborn
period with vesicles in a Blaschkoid distribution. A biopsy would show eosinophilic spongiosis with
dyskeratotic keratinocytes and pigment incontinence. A gene defect in NEMO has been identified as
the cause for the constellation of fingings


57) The most common location for this lesion would be:

A. Upper lip

B. Sole of footCorrect Choice

C. Buttock

D. Finger

E. Groin
Eccrine poroma is a benign, solitary tumor arsing in the lower portion of the epidermis. It is
characterized by small, uniformly cuboidal cells with deeply basophilic nuclei. The tumor mass is
assembled in broad anastomosing bands and may have narrow ductal lumina lined by eosinophilic
cuticle


58) The diagnosis is:
Image Link 1
Image Link 2
Image Link 3


A. microcystic adenexal carcinoma

B. breast carcinomaCorrect Choice

C. neuroendocrine carcinoma

D. tubular apocrine adenoma

E. infiltrative basal cell carcinoma
The diagnosis is metastatic breast carcinoma. In this example, there is a dense collection of cells
throughout the ermis. On closer examination, tehre are strands/cords of cells as well as some cells
arranged around lumina infiltrating through the dermis


59) Steatocystoma multiplex is associated with:

A. Jackson-Lawler Correct Choice

B. Jadassohn-Lewandowsky

C. Zinsser-Engman-Cole

D. Schaufer-Brunauer

E. Touraine-Solente-Gole
Jackson-Lawler (Jackson-Sertoli) is known as pachyonychia congenital type 2. Multiple steatocysts
can be seen in this condition


60) Weibel-Palade bodies are seen in:

A. Spitz Nevi

B. Endothelial cells Correct Choice

C. Cells infected with MCV



                                                                                                 16
D. Plasmacytoid Cells

E. Malakoplakia
Weibel-Palade bodies are seen in endothelial cells and are therefore found in vascular lesions.
Kamino bodies are found in Spitz nevi. Henderson Patterson bodies are seen in molluscum. Dutcher
bodies are intranuclear inclusions seen in plasmacytoid cells. Michaelis Gutmann bodies are partially
digested bacteria seen in malakoplakia


61) Eosinophilia-Myalgia syndrome is caused by:

A. Norwegian salt-petter

B. Unadultered Spanish grapeseed oil

C. Excessive anaerobic exercise

D. L-Tryptophan Correct Choice

E. Pb intoxication
The eosinophilia myalgia syndrome is characterized by marked peripheral eosinophilia with a clinical
spectrum of signs and symptoms, including generalized myalgias, pneumonitis, myocarditis,
neuropathy, encephalopathy and fibrosis. Many patients progress to a clinical picture clinically
indistinguishable from eosinophilic fasciitis. The disease is caused by the ingestion of certain lots of
L-tryptophan


62) Blue-gray pigmentation on the legs secondary to minocycline on biopsy stains with:

A. Fontana Masson and Perls Correct Choice

B. All of these answers are correct

C. Sudan black

D. Fontana Masson

E. Perls
There are three types of pigmentary change that are caused by minocycline. The blue-gray
pigmentation on the legs and the blue pigment in scars is thought to be secondary to a drug-protein
complex deposited in the dermis. The blue-gray pigment on the legs stains with Perls and Fontana-
Masson. The blue in scars (often on the face) stains with Perls. The muddy-brown discoloration on
sun-exposed areas shows increased basilar pigment and melanin incontinence on biopsy. It is likely
secondary to phototoxicity.


63) The diagnosis is:
Image Link 1
Image Link 2
Image Link 3


A. Artecoll reaction

B. goutCorrect Choice

C. mucinous carcinoma

D. Urbach-Wiethe's

E. Hunter's
In gout, there are characteristic amorphous light pink masses of material within which it is
sometimes possible to see outlines of needle-like spaces. The urate crystals can only be seen if




                                                                                                     17
alcohol fixation is used. The amorphous material is generally surrounded by histiocytes and foreign-
body giant cells


64) The histologic finding of "shoulder parakaratosis", parakeratosis with prediliection for the
follicular ostia, is characteristic of pityriasis rubra pilaris as well as:


A. Stasis dermatitis

B. Atopic dermatitis

C. Seborrheic dermatitisCorrect Choice

D. Nummular dermatitis

E. Allergic contact dermatitis
Parakeratosis refers to pyknotic keratinocyte nuclei in the stratum corneum, where nuclei are not
usually present. It is common in diseases with changes in the epidermis. Histologically seborrheic
dermatitis can shows "shoulder parakeratosis" with epidermal spongiosis. Histologically atopic,
nummular and contact dermatitis present with spongiosis with or without vesicles. Stasis dermatitis
presents with more dilated papillary dermal small blood vessels and hemosiderin


65) The diagnosis is:
Image   Link   1
Image   Link   2
Image   Link   3
Image   Link   4
Image   Link   5
Image   Link   6


A. scleroderma, early

B. erythema induratum

C. sarcoidosis

D. erythema nodosumCorrect Choice

E. subcutaneous granuloma annulare
In erythema nodoosum, one sees a predominantly septal panniculitis with some septal thickening
and fibrosis and an inflammatory infiltrate within the septae composed of lymphocytes, histiocytes,
eosinophils, and giant cells


66) The diagnosis is:

A. Psoriasis

B. Bowen's disease

C. Clear cell acanthoma Correct Choice

D. Trichilemmoma

E. Poroma
NEEDS EXPLANATIONS


67) Which type of artifact is shown here?

A. Electrodessication




                                                                                                   18
B. Dessication

C. Gel foamCorrect Choice

D. Microtome knife chatter

E. Freeze
The presence of deeply basophilic, wavy, angulated foreign material characteristic of gel foam
artifact


68) This is associated with MEN IIa:

A. Malignant peripheral nerve sheath tumor

B. Macular amyloidosis Correct Choice

C. Mucocele

D. Neurothekeoma

E. Chondroid syringoma
Macular amyloidosis is associated with MEN IIa


69) The diagnosis is:

A. Lymphoma

B. Small cell melanoma

C. Glomus tumor

D. Merkel cell carcinoma Correct Choice

E. Rhabdomyosarcoma
NEEDS EXPLANATIONS


70) The diagnosis is:
Image Link 1
Image Link 2
Image Link 3


A. acrospiroma

B. reticulated seborrheic keratosis

C. fibroepithelioma of PinkusCorrect Choice

D. syringofibroadenoma

E. tumor of the follicular infundibulum
In fibroepithelioma of Pinkus, there is a reticulated network of basaloid cells coming off of the
epidermis in a plate-like fashion. There is some peripheral palisading of cells. The cells are
embedded in a fibrotic stroma


71) Paraproteinemia is associated with all except:

A. Plane xanthoma

B. Necrobiotic xanthogranuloma




                                                                                                    19
C. Scleromyxedema

D. Sclerosing panniculitis Correct Choice

E. Scleredema
Sclerosing panniculitis (lipodermatosclerosis) displays characteristic changes in the fat
(lipomembranous change); it is not associated with paraproteinemia. Generalized plane xanthomas,
scleromyxedema, necrobiotic xanthogranuloma, scleredema, erythema elevatum diutinum,
xanthoma disseminatum, and pyoderma gangrenosum have all been associated with a
paraproteinemia


72) The diagnosis is:

A. Cylindroma

B. Trichoblastoma

C. Hidradenoma

D. Acrospiroma

E. Spiradenoma Correct Choice
NEEDS EXPLANATIONS


73) All of the following are true of reticulohistiocytoma except:

A. Trauma is precipitating factor

B. Rare occurrence in children

C. Association with arthritisCorrect Choice

D. Immunostaining is positive for OKM1

E. Giant cells with “ground-glass” cytoplasm
Reticulohistiocytomas, also called giant cell reticulohistiocytomas, occur almost exclusively in adults.
They are generally solitary, and unlike the multicentric type, are not associated with mutilating
arthritis or predisposition for malignancy


74) What is the diagnosis?

A. Spiradenoma

B. CylindromaCorrect Choice

C. Dermal duct tumor

D. Syringocystadenoma papilliferum

E. Trichoblastoma
This is a cylindroma, in which there is a jigsaw puzzle type arrangement of islands of basaloid cells
with intervening fibrous, pink stroma. The basaloid cells are sometimes rimmed by a thick, pink
basement membrane


75) Mulberry cells contain increased:

A. Mitochondria Correct Choice

B. Phagolysosomes and mitochondria




                                                                                                     20
C. Phagolysosomes

D. Golgi

E. Ribosomes
Hibernomas commonly arise in the neck, axillae, and posterior shoulder. The cells are
multivacuolated and resemble mulberries; the cells are filled with mitochondria, as are the cells in
normal brown fat


76) The endemic form of this disease may be transmited by:
Image Link 1
Image Link 2
Image Link 3


A. Glossina

B. SimuliumCorrect Choice

C. Lutzomyia

D. Phlebotomus

E. Triatoma
Triatoma (reduviid bug) species transmit American trypanosomiasis. Glossina is the genus of tsetse
flies that transmit African trypanosomiasis. Simulium is the genus of the black fly that can tranmit
Onchocerciasis and possibly the endemic form of pemphigus foliceus (fogo selvagem). Phlebotomus
and Lutzomyia are types of sandflies that can transmit Leishmaniasis, Carrion’s disease, and viral
sandfly fever


77) Which of the following hitologic features would be most helpful in differentiating lichenoid drug
eruption from lichen planus?


A. Squamatization of the basal layer

B. Band-like infiltrate with “Saw-tooth” rete ridges

C. Parakeratosis and eosinophilsCorrect Choice

D. Presence of pruritus

E. Civatte bodies
Lichenoid drug eruptions share clinical and histopathologic features with lichen planus. Sometimes
differentiation is not possible; however, eosinophil, parakeratosis, and a deeper perivascular
infiltrate is more suggestive of lichenoid drug. Implicated medications include captopril,
penicillamine, and chloroquine


78) In this patient, this test will be helpful in making the diagnosis:

A. Direct immunofluorescenceCorrect Choice

B. Tissue culture

C. Fluorescent antibody test for herpes

D. Patch test

E. KOH exam
This patient has penicillamine-induced pemphigus foliaceus. Direct immunofluorescence testing will
be very helpful as it should reveal intercellular antibodies within the epidermis




                                                                                                   21
79) This patient says the rash is spreading and not controlled with topical therapy. You give him a
course of oral treatment that lasts:


A. 1 week

B. 3 weeksCorrect Choice

C. 5 weeks

D. 4 weeks

E. 2 weeks
Generally, for poison ivy dermatitis, if patients are given a course of oral steroids, the course should
be at least 3 weeks long, as if the duration is shorter, patients may develop a rapid rebound


80) Langerhans cells express or are characterized by all of the following except:

A. HLA-DR

B. ChromagraninCorrect Choice

C. Birbeck granules

D. CD1a

E. S-100
Chromagranin stain neuroendocrine cells, Merkel cellcarcinomas and eccrine glands. They do not
stain Langerhans cells


81) What is the diagnosis?

A. Dermatomyofibroma

B. Leiomyosarcoma

C. Palisaded encapsulated neuromaCorrect Choice

D. Leiomyoma

E. Traumatic neuroma
This is a palisaded encapsulated neuroma, in which there are small bundles of cells that have wavy,
thin (elongated) nuclei and pink cytoplasm. The cells are separated by artifactual clefting. The
bundles are often located very superficially. Encapsulation is often incomplete/not obvious


82) This woman should have a workup for:

A. Nephrolithiasis

B. HemochromatosisCorrect Choice

C. Lymphoma

D. Thalassemia

E. Pancreatic cancer
Porphyria cutanea tarda has been shown to be associated with hemochromatosis. Patients with
porphyria cutanea tarda have mutations in the HFE gene, and early detection of mutations can
improve life expectancy for these patients




                                                                                                     22
83) The diagnosis is:
Image Link 1
Image Link 2
Image Link 3


A. myxedema

B. digital mucous cystCorrect Choice

C. reticulated erythematous mucinosis

D. mucinous granuloma annulare

E. papular mucinosis
In a digital mucous cyst, one sees a collection of mucin in the dermis beneath acral skin. This entity
is not a true cyst as there is no epithelial lining to the cyst. The mucin is largely composed of
hyaluronic acid


84) The diagnosis is:
Image   Link   1
Image   Link   2
Image   Link   3
Image   Link   4
Image   Link   5


A. neurothkeoma

B. palisaded encapsualted neuromaCorrect Choice

C. amelanotic blue nevus

D. dermatofibroma

E. leiomyoma
In palisaded encapsulated neuroma, there are broad fascicles of spindle cells set in a clear matrix.
The fascicles of spindle cells are sometimes clearly separated from the surrounding normal dermis
by a capsule, but other times blend into the dermis. The spindle cells have elongated, thin/tapered
nuclei. Palisading of nuclei is often not obvious


85) The diagnosis is:
Image Link 1
Image Link 2
Image Link 3


A. granuloma faciale

B. pigmented purpura

C. leukocytoclastic vasculitisCorrect Choice

D. mastocytosis

E. acrodermatitis of Mali
In leukocytoclastic vasculitis, on low power there is an inflammatory infiltrate generally clustered
around the vessels (although sometimes more dense and interstitial). There is extravasation of
erythrocytes around vessels with predominantly neutrophils around vessels and often within the
walls of the vessels. There is fragmentation of neutrophilic nuclei (“nuclear dust”) with fibrin (pink
amorphous material) within the walls of vessels and sometimes frank destruction of vessels


86) Which type of artifact is illustrated here?


                                                                                                     23
A. ElectrodessicationCorrect Choice

B. Gel foam

C. Knife chatter

D. Dessication

E. Freeze
The elogation of cells and spindling of nuclei with typical "string bean" appearance are characteristic
of electrodessication artifact


87) This patient’s biopsy will likely show:

A. Mononuclear cells with abundant cytoplasm around superficial vessels

B. Eosinophils at the dermoepidermal junction

C. Leukocytoclasia around superficial vessels

D. Lymphocytes at the dermoepidermal junctionCorrect Choice

E. Neutrophils at the dermoepidermal junction
This patient has erythema multiforme, and biopsy should show a lichenoid infiltrate of lymphocytes
at the dermoepidermal junction


88) The promontory sign is seen in:

A. Tufted angioma

B. Acroangiodermatitis of Mali

C. Kaposi’s sarcoma Correct Choice

D. Glomeruloid hemangioma

E. Spindle cell hemangioendothelioma
The promontory sign refers to the formation of new vessels around existing vessels and adnexal
structures. This is seen in Kaposi’s


89) What is the diagnosis?

A. Lupus erythematosus

B. Mycosis fungoides

C. Poroma

D. Porokeratosis

E. PsoriasisCorrect Choice
This is psoriasis, in which there is parakeratosis with entrapped neutrophils overlying a regularly
acanthotic epidermis. There is hypogranulosis of the epidermis with increased mitoses in the basal
layer. There are thinned suprapapillary plates with dilated vessels in the superficial dermal papillae.
There is a lympho-histiocytic infiltrate around superficial vessels. Occasionally, clusters of
neutrophils can be seen in the stratum spinosum (spongiform pustules of Kogoj


90) What is the diagnosis?

A. Melanoma



                                                                                                     24
B. Extramammary Paget’sCorrect Choice

C. Sebaceous carcinoma

D. Bowen’s

E. Condyloma
This is an example of extramammary Paget’s, in which there are atypical cells singly and in groups
within the epidermis. The cells have abundant bluish cytoplasm


91) This is secondary to:
Image Link 1
Image Link 2


A. paraproteinemia

B. a dull blade

C. silicone injections

D. gel foamCorrect Choice

E. metastatic carcinoma
Gel foam in tissue sections is a characteristic wavy material that stains bluish-gray


92) The predominant location of the cleft in transient neonatal pustular melanosis is:

A. Suprabasal

B. Basement membrane zone

C. Subcorneal/granularCorrect Choice

D. Dermal

E. Basal keratinocytes
Transient neonatal pustular melanosis is an idiopathic pustular eruption of newborns, mostly on the
chest, that heals with hyperpigmentation. It is most common on pigmented individuals.
Histologically it presents as subcorneal pustules with eosinophils and neutrophils


93) What is the diagnosis?

A. ChondrodermatitisCorrect Choice

B. Granular cell tumor

C. Bromoderma

D. Lichen simplex chronicus

E. Actinic keratosis
This is chondrodermatitis nodularis helices, in which there is hyperkeratosis and parakeratosis
overlying an altered/thickened epidermis. Beneath that area, in the dermis, there is often fibrosis.
Flanking the fibrosis on either side, there is a proliferation of vessels and inflammation (resembling
granulation tissue


94) A lichenoid infiltrate that surrounds eccrine glands is seen in:

A. Lichen planus




                                                                                                    25
B. Lichenoid purpura

C. Lichen striatusCorrect Choice

D. Lichenoid drug rection

E. Lichen planopilaris
Lichen striatus is an uncommon inflammatory dermatitis seen most commonly in children aged 5 to
15. It presents unilaterally along Blaschko's lines as raised, slightly scaly, erythematous papules,
which are often pruritic. These lesions typically regress spontaneously within a year. The
histopathologic features of lichen striatus include a superficial perivascular inflammatory
lymphohistiocytic infiltrate with rare plasma cells and eosinophils. There is a focal lichenoid infiltrate
in the papillary dermis with basilar vacuolar alteration and necrotic keratinocytes. Spongiosis with
exocytosis of lymphocytes can be seen in the epidermis. A specific and distinctive feature of lichen
striatus is the presence of an inflammatory infiltrate that surrounds hair follicles and eccrine glands


95) Similar follicles can be seen on skin from the:

A. EyelidCorrect Choice

B. Nose

C. Finger

D. Glans penis

E. Labia majora
Vellus hair follicles are seen commonly in accessory tragic and eyelid skin


96) What is the diagnosis?

A. Incontinentia pigmenti

B. Lichen striatus

C. Lichen simplex chronicus

D. Lichen planusCorrect Choice

E. Pityriasis lichenoides et varioliformis acuta
This is lichen planus, in which there is hyperkeratosis (and no parakeratosis), irregular acanthosis of
the epidermis, hypergranulosis (often in wedge shapes), saw-toothing of the basal layer, and a
band-like inflammatory infiltrate of predominantly lymphocytes (usually no eosinophils) at the
dermoepidermal junction. Occasionally, artifactual clefting can be seen at the dermoepidermal
junction (Max-Joseph space). Colloid bodies/Civatte bodies (amorphous pink material in globs) may
also be seen near the dermoepidermal junction


97) This reaction is most likely secondary to:
Image Link 1
Image Link 2


A. nicotinamide

B. tetracycline

C. mycophenolic acid

D. captoprilCorrect Choice

E. cyclosporine




                                                                                                       26
Bullous pemphigoid can be drug-induced and a common inciting drug is captopril. Other causes
include lasix, nalidixic acid, penicillamine, antibiotics (penicillin, amoxicillin, ampicillin), and PUVA


98) All have been associated with increased risk of breast cancer except:

A. Birt-Hogg-Dube Correct Choice

B. ataxia telangiectasia

C. Multicentric reticulohistiocytosis

D. Peutz-Jeghers

E. Cowden’s
Birt-Hogg-Dube is associated with renal cancer and thyroid cancer. Female carriers of a mutated
ATM (homozygous mutations ATM cause ataxia telangiectasia) have an increased risk of breast
cancer


99) The diagnosis is:
Image Link 1
Image Link 2
Image Link 3


A. microcystic adnexal carcinoma

B. mixed tumorCorrect Choice

C. syringofibroadenoma

D. mucinous carcinoma

E. papillary eccrine adenoma
Some authors separate mixed tumors into eccrine and apocrine types. In the eccrine mixed tumor
(pictured here), there are cords, clusters, and strands of basaloid cells forming lumina, some lined
by eosinophilic cuticles. These clusters of cells are embedded in a bluish myxoid/cartilaginous
stroma


100) The predominant location of the cleft in acropustulosis of infancy is:

A. Dermal

B. Basment mebrane zone

C. Subcorneal/granularCorrect Choice

D. Suprabasal

E. Basal keratinocytes
Acropustulosis of infancy presents as idiopathic pustules on acral skin. Diagnosis is made only after
other causes of pustules have been ruled out, and it usually resolves in a few years. The cleft in
acropustulosis of infancy is subcorneal/granular with neutrophils


101) Cicatricial pemphigoid antibodies directed against this are associated with high frequency of
malignancy:


A. Laminin 5 Correct Choice

B. Beta4-integrin

C. BPAg2



                                                                                                            27
D. Laminin 6

E. All of these answers are correct
Anti-laminin 5 cicatricial pemphigoid (CP) is also known as anti-epiligrin CP. Anti-epiligrin CP is
associated with an increased frequency of internal adenocarcinomas. Laminin 5 is composed of
three chains (heterotrimer), alpha3, beta3, gamma2. Antibodies are frequently directed against the
alpha3 chain, and so cross-reactivity can be observed with laminin 6, as laminin 6
(alpha3beta1gamma1) has the alpha3 chain as well. Beta4-integrin antibodies have been associated
with ocular CP. BPAg2 antibodies are seen in CP patients that have mucosal as well as skin disease


102) This patient may have antibodies to:
Image Link 1
Image Link 2
Image Link 3


A. metalloproteinase

B. transglutaminase

C. myeloperoxidaseCorrect Choice

D. proteinase-3

E. aminotransferase
The figures are consitent with polyarteritis nodosa. Patients with polyarteritis nodosa may have
antibodies to p-ANCA or myeloperoxidase


103) The diagnosis is:
Image   Link   1
Image   Link   2
Image   Link   3
Image   Link   4


A. malignant fibrous histiocytoma

B. nodular fascitisCorrect Choice

C. epithelioid sarcoma

D. neurofibroma

E. dermatofibrosarcoma protuberans
In nodular fasciitis, there is a ill-defined deep (often extending into fat) proliferation of plump
spindle cells that on higher power resemble “tissue-culture fibroblasts” with elongated cytoplasm
often set in a background of many small vessels and extravasated erythrocytes. The spindle cells
are arranged haphazardly, and the stroma is often myxoid. Mitoses are common


104) What is the diagnosis?

A. Psoriasis

B. Granular parakeratosisCorrect Choice

C. Lichen nitidus

D. Dermatophyte

E. Lichen planus




                                                                                                      28
Granular parakeratosis results from abnormal keratinization which generally occurs in the flexural
areas. Histologically, the thick parakeratotic layer with retention of keratohyaline granules. In
addition, the granular layer is preserved with relatively normal epidermis


105) What is the diagnosis?

A. Mastocytosis

B. Leprosy

C. Sarcoid

D. Lichenoid actinic keratosis

E. Lichen nitidusCorrect Choice
This is lichen nitidus, in which there is a “ball” of lymphocytes and histiocytes in the superficial
dermis abutting the epidermis surrounded on both sides by “claws” of the epidermis (rete


106) Which of the following drugs has been known to cause pyogenic granuloma?

A. CapecitabineCorrect Choice

B. Paclitaxel

C. Isosfamide

D. Daunorubicin

E. Mithramycin
Systemic retinoids, indinavir and capecitabine have all been describe to cause pyogenic granulomas


107) Which of the following histologic features is seen in aging skin?

A. Increased sebum production

B. Increased number of terminal hairs

C. Thickened dermal-epidermal junction

D. Fewer Langerhans cellsCorrect Choice

E. Increased mast cells
Histologic features of aging epidermis include flattened dermo-epidermal junction, occasional
nuclear atypia, decrease in the number of melanocytes and Langerhans cells. Changes that are
present in the dermis include atrophy, decrease in fibroblasts, mast cells and blood vessels


108) In this patient, this test will be helpful in making the diagnosis:

A. Patch test

B. Fluorescent antibody test for herpesCorrect Choice

C. Tissue culture

D. KOH exam

E. Indirect immunofluorescence
This patient has herpes zoster. A direct fluorescent antibody test for the varicella zoster virus can be
performed to confirm the diagnosis.




                                                                                                       29
109) This patient also has anemia. He needs screening for::
Image Link 1
Image Link 2
Image Link 3


A. malignancyCorrect Choice

B. liver cirrhosis

C. diabetes

D. pulmonary firbrosis

E. immunosuppression
The histology shows numerous neutrophils in the dermis with a lack of vasculitis, consistent with
Sweet’s syndrome. In a patient with Sweet’s syndrome, the presence of anemia is associated with
an internal malignancy.


110) Multiple such lesions are associated with::
Image Link 1
Image Link 2


A. Cowden'sCorrect Choice

B. Gorlin's

C. Werner's

D. Bloom's

E. Sipple's
Multiple sclerotic fibromas are seen in Cowden’s syndrome.


111) The diagnosis is:

A. Dermatofibrosarcoma

B. Angiolipoma Correct Choice

C. Epithelioid sarcoma

D. Nodular fasciitis

E. Liposarcoma
NEEDS EXPLANATIONS


112) Cellular neurothekeoma stains with:

A. Low molecular weight keratin

B. Stromelysin-3

C. Desmin

D. PGP-9.5 Correct Choice

E. S-100
PGP-9.5 and S100-a6 stains cellular neurothekeoma. Stromelysin-3 is positive in dermatofibromas
and negative in dermatofibrosarcoma protuberans. Desmin stains rhabdomyosarcoma. S-100 stains
neural tumors and melanocytic tumors among other things, but cellular neurothekeomas are
generally S100-negative



                                                                                                30
113) What stain may be used to differentiate this entity from metastatic oat cell carcinoma of the
lung?


A. HMB 45

B. CEA

C. PAS

D. Cytokeratin 20Correct Choice

E. S-100
Merkel cell carcinoma is a neuroendocrine cancer, usually of the head and neck. The tumor stain
with cytokeratin 20 which is expressed in a paranuclear dot-like pattern. This stain helps to
differentiate Merkel cell carcinoma from metastatic oat cell carinoma of the lung.


114) The diagnosis is:
Image Link 1
Image Link 2
Image Link 3


A. Epithelioid sarcoma

B. Angiosarcoma

C. Epithelioid hemangioendothelioma

D. Kaposi's sarcomaCorrect Choice

E. Aneurismal dermatofibroma
In nodular Kaposi’s sarcoma, one sees a proliferation of spindle cells, often arranged in nodules
separated by fibrous bands. On higher power examination of the spindle cells, numerous
extravasated erythrocytes can be seen between the cells. Often, hemosiderin-filled macrophages
and plasma cells can be seen as well. The spindle cells are packed closely together and often will
have intracytoplasmic pink inclusions (erythrophagolysosomes


115) The diagnosis is:

A. Psoriasis

B. Pityriasis rubra pilarisCorrect Choice

C. Inflammatory linear verrucous epidermal nevus

D. Prurigo nodularis

E. Ichthyosis
NEEDS EXPLANATION


116) What is the diagnosis?

A. Dilated poreCorrect Choice

B. Fibrofolliculoma

C. Pilar sheath acanthoma

D. Keratosis pilaris




                                                                                                     31
E. Trichoadenoma
This is a dilated pore, in which there is an invagination lined by epidermis that is slightly acanthotic


117) These cells should stain with:
Image Link 1
Image Link 2
Image Link 3


A. S100

B. factor XIIIa

C. cytokeratin

D. CD34

E. actinCorrect Choice
This is a dermatomyofibroma. Dermatomyofibromas are often found over the scapula of women.
The spindle cells are oriented parallel to the epidermis, and stain with vimentin and non-specific
muscle actin. The spinde cells do not stain with desmin, S100, CD34, or Factor XIIIa


118) What is the diagnosis?

A. Impetigo

B. Bullous pemphigoidCorrect Choice

C. Subcutaneous lupus erythematosus

D. Dermatitis herpetiformis

E. Polymorphous light eruption
This is bullous pemphigoid, in which a subepidermal bullae/vesicle displays numerous eosinophils
lining up at the dermo-epidermal junction


119) This developed in a patient with a history of breast cancer s/p surgical excision/radiation. Her
diagnosis is:
Image Link 1
Image Link 2
Image Link 3


A. Horner's

B. Stewart-TrevesCorrect Choice

C. Parkes-Weber

D. Kettle's

E. Klippel-Trenaunay
Angiosarcoma can develop in a lymphedematous extremity. When it develops in the upper
extremity after surgical treatment of breast cancer, it is referred to as Stewart-Treves syndrome.
When it develops in the lower extremity after lymph node dissection for a melanoma, it is referred
to as Kettle's syndrome


120) What is the diagnosis?

A. Trichofolliculoma




                                                                                                      32
B. Spiradenoma

C. Trichilemmoma

D. Mixed tumor

E. TrichoepithliomaCorrect Choice
This is a trichoepithelioma, in which there are islands of basaloid cells in a somewhat fibrous stroma
with no retraction between the islands and the stroma. Often horn cysts are seen (not shown).


121) The inclusions in infantile digital fibromatosis stain for trichrome and:

A. Thioflavin T

B. Pentahydroxy flavanol

C. Osmium tetroxide

D. Phosphotungstic acid hematoxylin Correct Choice

E. Bodian
Osmium tetroxide stains fat. Thioflavin T stains amyloid. The Bodian stain is for nerves.
Pentahydroxy flavanol is a fluorescent stain for calcium


122) The diagnosis is:
Image   Link   1
Image   Link   2
Image   Link   3
Image   Link   4


A. myofibromatosis

B. schwannoma

C. Kaposi's sarcoma

D. neurofibromaCorrect Choice

E. leiomyoma
This is a plexiform neurofibroma. There are discrete nodules of spindle cells within the dermis. On
higher power, the cells have wavy nuclei with pink cytoplasm


123) What is the diagnosis?

A. Carbon tattooCorrect Choice

B. Monsel’s reaction

C. Blue nevus

D. Postinflammatory hyperpigmentation

E. Minocycline-induced hyperpigmentation
This is a carbon tattoo, in which there are extracellular and intracellular particles of black material in
the superficial dermis


124) What is the diagnosis?

A. Deep penetrating nevus




                                                                                                       33
B. Recurrent nevus

C. Congenital nevusCorrect Choice

D. Epithelioid blue nevus

E. Nevoid melanoma
This is a congenital nevus, in which there are nests of nevomelanocytic cells at the dermoepidermal
junction and extending deep into the dermis. In the deeper dermis, the cells infiltrate through
collagen bundles and extend around adnexal structures. There is hyperkeratosis, acanthosis, and
papillomatosis of the surface epidermis


125) The green color in chloroma is secondary to:

A. Stromelysin

B. Myeloperoxidase Correct Choice

C. Fumarase

D. Chloracetate

E. Alkaline phosphatase
Chloromas are greenish tumor grossly secondary to involvement of the skin in acute granulocytic
leukemia. The green color is secondary to myeloperoxidase


126) Supporting evidence for the diagnosis of mycosis fungoides is CD4+ lymphocytes with loss of
CD7 as well as loss of


A. CD20

B. CD2

C. CD30

D. CD5 Correct Choice

E. CD3
CD5 as well as CD7 are sometimes lost on the surface of epidermotropic T cells in mycosis
fungoides. CD2, CD3, and CD5 are T cell markers. CD20 is a B cell marker. CD30 is positive in
anaplastic large cell lymphoma cells, Hodgkin’s lymphoma, and lymphomatoid papulosis. Reactive
infiltrates can also have some CD30-positive cells


127) Clinically, a nondescript hyperkeratotic papule on the ulnar side of the base of the fifth finger
is most likely:


A. Cutaneous horn

B. Accessory digit Correct Choice

C. Digital fibromatosis

D. Acquired digital fibrokeratoma

E. Glomus tumor
Accessory digits (supernumerary digits) are usually found at the base of the fifth finger, often
bilaterally


128) Multiple trichoepitheliomas are seen in all except:



                                                                                                    34
A. Brooke-Fordyce syndrome

B. Rombo syndrome

C. Gorlin's syndrome Correct Choice

D. Brooke-Spiegler syndrome

E. Bazex's syndrome
Gorlin's syndrome is nevoid basal cell carcinoma syndrome; multiple trichoepitheliomas are not
seen. Several syndromes have been associated with multiple trichoepitheliomas: Basex, Brooke-
Fordyce, Brooke-Spiegler, Rombo, and possibly Rasmussen. (Rasmussen described one family in
1975 with autosomal dominant inheritance of multiple trichoepitheliomas, milia, and cylindromas.)
Basex syndrome (follicular atrophoderma, hypotrichosis, occasional trichoepitheliomas, basal cell
carcinomas, and localized or generalized hypohidrosis) is inherited in an X-linked dominant manner.
Brooke and Fordyce both described multiple trichoepitheliomas concurrently in 1892, and therefore
multiple familial trichoepitheliomas are sometimes called “Brooke-Fordyce” sydrome. Spiegler
described patients with multiple cylindromas in 1899 and also noted that many of these patients
had mutiple trichoepitheliomas; more recently it has been noted that multiple spiradenomas may be
seen in patients with multiple trichoepitheliomas and cylinidromas; this co-occurrence of tumors has
been referred to as “Brooke-Spiegler” syndrome. (Brooke-Fordyce and Brooke-Spiegler are likely
the same syndrome.) Rombo syndrome is characterized by vermiculate atrophoderma, multiple
BCCs, multiple trichoepitheliomas, cyanosis and peripheral vasodilation


129) The diagnosis is::
Image   Link   1
Image   Link   2
Image   Link   3
Image   Link   4


A. hypertrophic scarCorrect Choice

B. leiomyoma

C. dermatomyofibroma

D. neurofibroma

E. dermatofibroma


130) What is the diagnosis?

A. Adenoid cystic carcinoma

B. Trichoblastoma

C. Microcystic adnexal carcinoma

D. Trichodiscoma

E. Morpheaform basal cell carcinomaCorrect Choice
This is a morpheaform basal cell carcinoma, in which there are very infiltrative islands of basaloid
cells, extending deep into the dermis. Around some of the basaloid islands, there is new, pink
collagen. The basaloid proliferation off of the surface of the epidermis is more typical of a superficial
multicentric basal cell carcinoma and aids in the diagnosis


131) This patient should be examined for::
Image   Link   1
Image   Link   2
Image   Link   3
Image   Link   4



                                                                                                      35
A. photosensitivity

B. keratoacanthomasCorrect Choice

C. pigmentary anomalies

D. odontogenic cysts

E. arsenical keratoses
The figures show a sebaceous adenoma. Sebaceous adenomas are associated with Muir-Torre
syndrome, in which patients can have an internal malignancy and multiple keratoacanthomas


132) The most likely diagnosis for this lesion would be:

A. Verruca

B. Acrochordon

C. Acquired digital fibrokeratomaCorrect Choice

D. Amputation neuroma

E. Supernumery digit
The diagnosis of this acral lesion is an acquired digital fibrokeratoma which shows a small, exophytic
circumscribed lesions. Collagen bundles are oriented perpendicularly to the skin surface. The lesion
lacks nerve twigs and bone, which may be present in supernumery digit or amputation neuroma


133) Clear cell syringomas are associated with:

A. Sarcoidosis

B. Malignancy

C. Diabetes Correct Choice

D. Lichen myxedematosis

E. Argyria
Clear cell syringomas are associated with diabetes. Syringomas are associated with Down’s
syndrome


134) The diagnosis is:
Image Link 1
Image Link 2
Image Link 3


A. tubular apocrine adenoma

B. dermal duct tumor

C. syringoma

D. trichoepithelioma

E. adenoid cystic carcinomaCorrect Choice
In adenoid cystic carcinoma, there are cords and tubules of basaloid cells, arranged in a cribiform
pattern. The cystic spaces contain a bluish material. Perineural invasion is common. The cells stain
positively for epithelial membrane antigen (EMA).


135) What is the cause of this pigmentary condition?


                                                                                                   36
A. Hemochromatosis

B. Post-inflammatory hyperpigmetation

C. Minocycline ingestion

D. ArgyriaCorrect Choice

E. Chrysiasis
The characteristic feature of argyria is the presence of black granules in the eccrine glands. It
differentiates this condition from other pigmentary disorders


136) The diagnosis is:
Image   Link   1
Image   Link   2
Image   Link   3
Image   Link   4


A. Erythropoietic protoporphyria

B. Lipoid proteinosis

C. Radiation dermatitis

D. Colloid milium

E. Lichen sclerosus et atrophicusCorrect Choice
In lichen sclerosus et atrophicus, there is often an atrophic epidermis with overlying orthokeratosis
that is thicker than the stratum spinosum, with some follicular plugging. There is sometimes a
subepidermal separation. The upper dermis is homogenized and pink/pale. Sometimes underlying
the homogenized zone, there is a band-like infiltrate (not seen here


137) A patient with a blue-red discoloration of the nail plate reports that the same finger becomes
very tender when exposed to the cold. You suspect a:


A. Pyogenic granuloma

B. Glomus tumorCorrect Choice

C. Periungual verruca

D. Mucous cyst

E. Pterygium
Glomus tumors are tumors of the arterio-venous anastamosis of the digital dermis. They occur most
frequently in the nail bed. The commonly have a bluish-red discoloration and may be tender or
painful with exposure to heat or cold


138) Verruciform xanthoma is seen most commonly on:

A. Distal extremities

B. Mucosal surfaces and trunk/proximal extremities

C. Oral mucosae and genital areas Correct Choice

D. Nail bed and periungual areas

E. Head and neck
NEEDS EXPLANATION




                                                                                                    37
139) This patient developed an acute vesicular rash after eating a mango. She has returned for a
routine follow-up. She needs to be careful of exposure to:


A. All of these answers are correct

B. Ginkgo fruitCorrect Choice

C. Croton

D. Ragweed

E. Tea tree oil
Patients allergic to the peel of a mango can also be allergic to other plants/products of the
Anacardiaceae family. Cross-reactions can occur with exposure to any plants of the genus
Toxicodendron, to the oil from the cashew nut shell, to the Brazilian pepper tree, to lacquer from
the Japanese lacquer tree, to ink from the Indian marking nut, and to the fruit pulp of the ginkgo
tree, and others


140) The diagnosis is:
Image   Link   1
Image   Link   2
Image   Link   3
Image   Link   4


A. Blastomycosis

B. Cryptococcosis

C. Histoplasmosis

D. Toxoplasmosis

E. LeishmaniaisisCorrect Choice
The diagnosis is Leishmaniasis. In this condition, the epidermis is often ulcerated (not seen in this
case) with a dense infiltrate is seen within the dermis. There are numerous macrophages
(Leishman-Donovan bodies) within the infiltrate that have intracellular amastigotes within them. On
close examination of the parasites, a kinetoplast is evident. In contrast to histoplasmosis, a discrete
halo is not seen around the amastigotes


141) Caterpillar bodies are seen in:

A. Dyskeratosis congenital

B. Porphyria cutanea tarda Correct Choice

C. Amyloidosis

D. Lipoid proteinosis

E. Mucocele
Caterpillar bodies are thought to be type IV collagen


142) A healthy 6 month old girl has a subcutaneous nodule above her right eyebrow. A skin biopsy
demonstrates a cystic lesion with adnexal structures in the wall. Your diagnosis is:


A. Nevus sebaceous

B. Epidermal inclusion cyst

C. Steatocystoma




                                                                                                     38
D. Pilar cyst

E. Dermoid cystCorrect Choice
Dermoid cysts present along lines of embryonic closure. The are most commonly found on the head
(around the eyes) and the neck. They are lined by an epidermis that contains various epidermal
appendages that are usually fully matured


143) The diagnosis is:
Image Link 1
Image Link 2
Image Link 3


A. basaloid squamous cell carcinoma

B. malignant acrospiroma

C. poroid squamous cell carcinoma

D. porocarcinoma

E. basosquamous carcinomaCorrect Choice
In basosquamous carcinomas, there are areas that appear typical of basal cell carcinoma (with
basaloid cells coming off the epidermis in buds with peripheral palisading) as well as areas typical of
squamous cell carcinoma (with atypical keratinocytes that are more pink and angular than basaloid
cells). Often, ulcerated basal cell carcinomas have squamous differentiation at the base of the ulcer
and this is not to be confused with a basosquamous carcinoma, in which there are areas of both
basal cell carcinoma and squamous cell carcinoma


144) The diagnosis is:

A. Dermatofibroma Correct Choice

B. Dermatomyofibroma

C. Neurofibroma

D. Plexiform fibrohistiocytic tumor

E. Infantile digital fibromatosis
NEEDS EXPLANATIONS




                                                                                                    39

More Related Content

What's hot

Immunofluorescence in dermatopathology
Immunofluorescence in dermatopathologyImmunofluorescence in dermatopathology
Immunofluorescence in dermatopathologyNeha Sharma
 
ETAS_MCQ_03 a genodermatoses
ETAS_MCQ_03 a genodermatosesETAS_MCQ_03 a genodermatoses
ETAS_MCQ_03 a genodermatosesDerma202
 
ETAS_MCQ_16 dermatological drugs
ETAS_MCQ_16 dermatological drugsETAS_MCQ_16 dermatological drugs
ETAS_MCQ_16 dermatological drugsDerma202
 
ETAS_MCQ_01 structures of skin
ETAS_MCQ_01 structures of skinETAS_MCQ_01 structures of skin
ETAS_MCQ_01 structures of skinDerma202
 
Epidermal kinetics
Epidermal kineticsEpidermal kinetics
Epidermal kineticsRohit Singh
 
Dermo epidermal junction
Dermo epidermal junctionDermo epidermal junction
Dermo epidermal junctionHimani tandon
 
Direct Immunofluorescence in Dermatology
Direct Immunofluorescence in DermatologyDirect Immunofluorescence in Dermatology
Direct Immunofluorescence in DermatologyJerriton Brewin
 
Immunofluorescence in dermatopathology
Immunofluorescence in dermatopathologyImmunofluorescence in dermatopathology
Immunofluorescence in dermatopathologySumit Mandal
 
Cutaneous pseudolymphoma
Cutaneous pseudolymphomaCutaneous pseudolymphoma
Cutaneous pseudolymphomadrtousif
 
Dermatology board review
Dermatology board reviewDermatology board review
Dermatology board reviewAhmed Amer
 
Epidermopoeisis - development of skin
Epidermopoeisis - development of skin Epidermopoeisis - development of skin
Epidermopoeisis - development of skin Kriti Maheshwari
 
MCQ August with answers - Dr Ameen Alawadhi
MCQ August with answers - Dr Ameen AlawadhiMCQ August with answers - Dr Ameen Alawadhi
MCQ August with answers - Dr Ameen Alawadhiaskadermatologist
 

What's hot (20)

Immunofluorescence in dermatopathology
Immunofluorescence in dermatopathologyImmunofluorescence in dermatopathology
Immunofluorescence in dermatopathology
 
ETAS_MCQ_03 a genodermatoses
ETAS_MCQ_03 a genodermatosesETAS_MCQ_03 a genodermatoses
ETAS_MCQ_03 a genodermatoses
 
ETAS_MCQ_16 dermatological drugs
ETAS_MCQ_16 dermatological drugsETAS_MCQ_16 dermatological drugs
ETAS_MCQ_16 dermatological drugs
 
Histoid leprosy
Histoid leprosyHistoid leprosy
Histoid leprosy
 
Cutaneous pseudolymphoma
Cutaneous pseudolymphomaCutaneous pseudolymphoma
Cutaneous pseudolymphoma
 
ETAS_MCQ_01 structures of skin
ETAS_MCQ_01 structures of skinETAS_MCQ_01 structures of skin
ETAS_MCQ_01 structures of skin
 
Ganyang MCQ Dermatology
Ganyang MCQ DermatologyGanyang MCQ Dermatology
Ganyang MCQ Dermatology
 
Epidermal kinetics
Epidermal kineticsEpidermal kinetics
Epidermal kinetics
 
Dermo epidermal junction
Dermo epidermal junctionDermo epidermal junction
Dermo epidermal junction
 
Lichen planus ppt
Lichen planus pptLichen planus ppt
Lichen planus ppt
 
Derma.
Derma.Derma.
Derma.
 
Direct Immunofluorescence in Dermatology
Direct Immunofluorescence in DermatologyDirect Immunofluorescence in Dermatology
Direct Immunofluorescence in Dermatology
 
Immunofluorescence in dermatopathology
Immunofluorescence in dermatopathologyImmunofluorescence in dermatopathology
Immunofluorescence in dermatopathology
 
Cutaneous pseudolymphoma
Cutaneous pseudolymphomaCutaneous pseudolymphoma
Cutaneous pseudolymphoma
 
Skin tumors
Skin tumorsSkin tumors
Skin tumors
 
Dermatology board review
Dermatology board reviewDermatology board review
Dermatology board review
 
Epidermopoeisis - development of skin
Epidermopoeisis - development of skin Epidermopoeisis - development of skin
Epidermopoeisis - development of skin
 
Merkel cells
Merkel cellsMerkel cells
Merkel cells
 
MCQ August with answers - Dr Ameen Alawadhi
MCQ August with answers - Dr Ameen AlawadhiMCQ August with answers - Dr Ameen Alawadhi
MCQ August with answers - Dr Ameen Alawadhi
 
Adnexal Neoplasms
Adnexal NeoplasmsAdnexal Neoplasms
Adnexal Neoplasms
 

Viewers also liked

ETAS_MCQ_15 dermatologic and cosmetic surgery
ETAS_MCQ_15 dermatologic and cosmetic surgeryETAS_MCQ_15 dermatologic and cosmetic surgery
ETAS_MCQ_15 dermatologic and cosmetic surgeryDerma202
 
dermatology.1 eryth, telan, urt & hshp(dr.faraydwn)
dermatology.1 eryth, telan, urt & hshp(dr.faraydwn)dermatology.1 eryth, telan, urt & hshp(dr.faraydwn)
dermatology.1 eryth, telan, urt & hshp(dr.faraydwn)student
 
Dermatology osce slides
Dermatology osce slidesDermatology osce slides
Dermatology osce slidesHabrol Afzam
 
ETAS_MCQ_10 manifestations of systemic diseases1
ETAS_MCQ_10 manifestations of systemic diseases1ETAS_MCQ_10 manifestations of systemic diseases1
ETAS_MCQ_10 manifestations of systemic diseases1Derma202
 
ETAS_MCQ_11 disorder of hair and nails
ETAS_MCQ_11 disorder of hair and nailsETAS_MCQ_11 disorder of hair and nails
ETAS_MCQ_11 disorder of hair and nailsDerma202
 
Dermatology without pics
Dermatology without picsDermatology without pics
Dermatology without picsess_online
 

Viewers also liked (8)

ETAS_MCQ_15 dermatologic and cosmetic surgery
ETAS_MCQ_15 dermatologic and cosmetic surgeryETAS_MCQ_15 dermatologic and cosmetic surgery
ETAS_MCQ_15 dermatologic and cosmetic surgery
 
dermatology.1 eryth, telan, urt & hshp(dr.faraydwn)
dermatology.1 eryth, telan, urt & hshp(dr.faraydwn)dermatology.1 eryth, telan, urt & hshp(dr.faraydwn)
dermatology.1 eryth, telan, urt & hshp(dr.faraydwn)
 
Dermatology osce slides
Dermatology osce slidesDermatology osce slides
Dermatology osce slides
 
ETAS_MCQ_10 manifestations of systemic diseases1
ETAS_MCQ_10 manifestations of systemic diseases1ETAS_MCQ_10 manifestations of systemic diseases1
ETAS_MCQ_10 manifestations of systemic diseases1
 
ETAS_MCQ_11 disorder of hair and nails
ETAS_MCQ_11 disorder of hair and nailsETAS_MCQ_11 disorder of hair and nails
ETAS_MCQ_11 disorder of hair and nails
 
Dermatology without pics
Dermatology without picsDermatology without pics
Dermatology without pics
 
Fe
FeFe
Fe
 
Serofuloderma
SerofulodermaSerofuloderma
Serofuloderma
 

Similar to Dermatopathology Generator

Krok 1 - 2015 (Path-Anatomy)
Krok 1 - 2015 (Path-Anatomy)Krok 1 - 2015 (Path-Anatomy)
Krok 1 - 2015 (Path-Anatomy)Eneutron
 
Previous year question on staining based on neet pg, usmle, plab and fmge or ...
Previous year question on staining based on neet pg, usmle, plab and fmge or ...Previous year question on staining based on neet pg, usmle, plab and fmge or ...
Previous year question on staining based on neet pg, usmle, plab and fmge or ...Abhishek Gupta
 
Krok 1 - 2014 (Path-Anatomy)
Krok 1 - 2014 (Path-Anatomy)Krok 1 - 2014 (Path-Anatomy)
Krok 1 - 2014 (Path-Anatomy)Eneutron
 
Krok 1 - 2014 Path-Anatomy Base (General Medicine)
Krok 1 - 2014 Path-Anatomy Base (General Medicine)Krok 1 - 2014 Path-Anatomy Base (General Medicine)
Krok 1 - 2014 Path-Anatomy Base (General Medicine)E_neutron
 
Krok 1 2014 - path anatomy
Krok 1   2014 - path anatomyKrok 1   2014 - path anatomy
Krok 1 2014 - path anatomyEneutron
 
23204907
2320490723204907
23204907radgirl
 
Previous year question on medulloblastoma based on neet pg, usmle, plab and f...
Previous year question on medulloblastoma based on neet pg, usmle, plab and f...Previous year question on medulloblastoma based on neet pg, usmle, plab and f...
Previous year question on medulloblastoma based on neet pg, usmle, plab and f...Medico Apps
 
23204961
2320496123204961
23204961radgirl
 
Basal cell Adenoma and Canalicular Adenoma Doctor Faris Alabeedi MSc, MMedSc,...
Basal cell Adenoma and Canalicular Adenoma Doctor Faris Alabeedi MSc, MMedSc,...Basal cell Adenoma and Canalicular Adenoma Doctor Faris Alabeedi MSc, MMedSc,...
Basal cell Adenoma and Canalicular Adenoma Doctor Faris Alabeedi MSc, MMedSc,...Doctor Faris Alabeedi
 
Breast Atypical Lesions
Breast Atypical LesionsBreast Atypical Lesions
Breast Atypical Lesionskurmuk
 
Previous year question on lichen planus based on neet pg, usmle, plab and fmg...
Previous year question on lichen planus based on neet pg, usmle, plab and fmg...Previous year question on lichen planus based on neet pg, usmle, plab and fmg...
Previous year question on lichen planus based on neet pg, usmle, plab and fmg...Abhishek Gupta
 
Previous year question on lichen planus based on neet pg, usmle, plab and fmg...
Previous year question on lichen planus based on neet pg, usmle, plab and fmg...Previous year question on lichen planus based on neet pg, usmle, plab and fmg...
Previous year question on lichen planus based on neet pg, usmle, plab and fmg...Abhishek Gupta
 
neetpg LRR PART2 (for march24).pdf
neetpg LRR PART2 (for march24).pdfneetpg LRR PART2 (for march24).pdf
neetpg LRR PART2 (for march24).pdfShubhamRoy947322
 
Krok 1 - 2007 Question Paper (Stomatology)
Krok 1 - 2007 Question Paper (Stomatology)Krok 1 - 2007 Question Paper (Stomatology)
Krok 1 - 2007 Question Paper (Stomatology)Eneutron
 
23204955
2320495523204955
23204955radgirl
 
Krok 1 - 2008 Question Paper (Stomatology)
Krok 1 - 2008 Question Paper (Stomatology)Krok 1 - 2008 Question Paper (Stomatology)
Krok 1 - 2008 Question Paper (Stomatology)Eneutron
 
Krok1 stomatology - 2016
Krok1   stomatology - 2016Krok1   stomatology - 2016
Krok1 stomatology - 2016Eneutron
 

Similar to Dermatopathology Generator (20)

Krok 1 - 2015 (Path-Anatomy)
Krok 1 - 2015 (Path-Anatomy)Krok 1 - 2015 (Path-Anatomy)
Krok 1 - 2015 (Path-Anatomy)
 
file
file file
file
 
Previous year question on staining based on neet pg, usmle, plab and fmge or ...
Previous year question on staining based on neet pg, usmle, plab and fmge or ...Previous year question on staining based on neet pg, usmle, plab and fmge or ...
Previous year question on staining based on neet pg, usmle, plab and fmge or ...
 
Krok 1 - 2014 (Path-Anatomy)
Krok 1 - 2014 (Path-Anatomy)Krok 1 - 2014 (Path-Anatomy)
Krok 1 - 2014 (Path-Anatomy)
 
Krok 1 - 2014 Path-Anatomy Base (General Medicine)
Krok 1 - 2014 Path-Anatomy Base (General Medicine)Krok 1 - 2014 Path-Anatomy Base (General Medicine)
Krok 1 - 2014 Path-Anatomy Base (General Medicine)
 
Krok 1 2014 - path anatomy
Krok 1   2014 - path anatomyKrok 1   2014 - path anatomy
Krok 1 2014 - path anatomy
 
23204907
2320490723204907
23204907
 
Mcqs for Ophthal pgs 1
Mcqs  for Ophthal pgs 1Mcqs  for Ophthal pgs 1
Mcqs for Ophthal pgs 1
 
Previous year question on medulloblastoma based on neet pg, usmle, plab and f...
Previous year question on medulloblastoma based on neet pg, usmle, plab and f...Previous year question on medulloblastoma based on neet pg, usmle, plab and f...
Previous year question on medulloblastoma based on neet pg, usmle, plab and f...
 
23204961
2320496123204961
23204961
 
Basal cell Adenoma and Canalicular Adenoma Doctor Faris Alabeedi MSc, MMedSc,...
Basal cell Adenoma and Canalicular Adenoma Doctor Faris Alabeedi MSc, MMedSc,...Basal cell Adenoma and Canalicular Adenoma Doctor Faris Alabeedi MSc, MMedSc,...
Basal cell Adenoma and Canalicular Adenoma Doctor Faris Alabeedi MSc, MMedSc,...
 
Breast Atypical Lesions
Breast Atypical LesionsBreast Atypical Lesions
Breast Atypical Lesions
 
Previous year question on lichen planus based on neet pg, usmle, plab and fmg...
Previous year question on lichen planus based on neet pg, usmle, plab and fmg...Previous year question on lichen planus based on neet pg, usmle, plab and fmg...
Previous year question on lichen planus based on neet pg, usmle, plab and fmg...
 
Previous year question on lichen planus based on neet pg, usmle, plab and fmg...
Previous year question on lichen planus based on neet pg, usmle, plab and fmg...Previous year question on lichen planus based on neet pg, usmle, plab and fmg...
Previous year question on lichen planus based on neet pg, usmle, plab and fmg...
 
neetpg LRR PART2 (for march24).pdf
neetpg LRR PART2 (for march24).pdfneetpg LRR PART2 (for march24).pdf
neetpg LRR PART2 (for march24).pdf
 
Krok 1 - 2007 Question Paper (Stomatology)
Krok 1 - 2007 Question Paper (Stomatology)Krok 1 - 2007 Question Paper (Stomatology)
Krok 1 - 2007 Question Paper (Stomatology)
 
23204955
2320495523204955
23204955
 
Krok 1 - 2008 Question Paper (Stomatology)
Krok 1 - 2008 Question Paper (Stomatology)Krok 1 - 2008 Question Paper (Stomatology)
Krok 1 - 2008 Question Paper (Stomatology)
 
Congenital myofibroma
Congenital myofibromaCongenital myofibroma
Congenital myofibroma
 
Krok1 stomatology - 2016
Krok1   stomatology - 2016Krok1   stomatology - 2016
Krok1 stomatology - 2016
 

More from Derma202

Phototherapy treatment protocol
Phototherapy treatment protocolPhototherapy treatment protocol
Phototherapy treatment protocolDerma202
 
Histopathplogical photos
Histopathplogical photosHistopathplogical photos
Histopathplogical photosDerma202
 
Slide study from ETAS
Slide  study from ETASSlide  study from ETAS
Slide study from ETASDerma202
 
Arab board primary exam in dermatology 2012
Arab board primary exam  in dermatology 2012Arab board primary exam  in dermatology 2012
Arab board primary exam in dermatology 2012Derma202
 
Dermatology
DermatologyDermatology
DermatologyDerma202
 
ETAS_MCQ_14 plants and creatures of dermatologic significance
ETAS_MCQ_14 plants and creatures of dermatologic significanceETAS_MCQ_14 plants and creatures of dermatologic significance
ETAS_MCQ_14 plants and creatures of dermatologic significanceDerma202
 
ETAS_MCQ_13 photobiology and photosensitivity disorders
ETAS_MCQ_13 photobiology and photosensitivity disordersETAS_MCQ_13 photobiology and photosensitivity disorders
ETAS_MCQ_13 photobiology and photosensitivity disordersDerma202
 
ETAS_MCQ_03 b genodermatoses
ETAS_MCQ_03 b genodermatosesETAS_MCQ_03 b genodermatoses
ETAS_MCQ_03 b genodermatosesDerma202
 
Derm handbook for medical students and junior doctors 2010
Derm handbook for medical students and junior doctors 2010Derm handbook for medical students and junior doctors 2010
Derm handbook for medical students and junior doctors 2010Derma202
 

More from Derma202 (9)

Phototherapy treatment protocol
Phototherapy treatment protocolPhototherapy treatment protocol
Phototherapy treatment protocol
 
Histopathplogical photos
Histopathplogical photosHistopathplogical photos
Histopathplogical photos
 
Slide study from ETAS
Slide  study from ETASSlide  study from ETAS
Slide study from ETAS
 
Arab board primary exam in dermatology 2012
Arab board primary exam  in dermatology 2012Arab board primary exam  in dermatology 2012
Arab board primary exam in dermatology 2012
 
Dermatology
DermatologyDermatology
Dermatology
 
ETAS_MCQ_14 plants and creatures of dermatologic significance
ETAS_MCQ_14 plants and creatures of dermatologic significanceETAS_MCQ_14 plants and creatures of dermatologic significance
ETAS_MCQ_14 plants and creatures of dermatologic significance
 
ETAS_MCQ_13 photobiology and photosensitivity disorders
ETAS_MCQ_13 photobiology and photosensitivity disordersETAS_MCQ_13 photobiology and photosensitivity disorders
ETAS_MCQ_13 photobiology and photosensitivity disorders
 
ETAS_MCQ_03 b genodermatoses
ETAS_MCQ_03 b genodermatosesETAS_MCQ_03 b genodermatoses
ETAS_MCQ_03 b genodermatoses
 
Derm handbook for medical students and junior doctors 2010
Derm handbook for medical students and junior doctors 2010Derm handbook for medical students and junior doctors 2010
Derm handbook for medical students and junior doctors 2010
 

Recently uploaded

Call Girls Whitefield Just Call 7001305949 Top Class Call Girl Service Available
Call Girls Whitefield Just Call 7001305949 Top Class Call Girl Service AvailableCall Girls Whitefield Just Call 7001305949 Top Class Call Girl Service Available
Call Girls Whitefield Just Call 7001305949 Top Class Call Girl Service Availablenarwatsonia7
 
Call Girls In Andheri East Call 9920874524 Book Hot And Sexy Girls
Call Girls In Andheri East Call 9920874524 Book Hot And Sexy GirlsCall Girls In Andheri East Call 9920874524 Book Hot And Sexy Girls
Call Girls In Andheri East Call 9920874524 Book Hot And Sexy Girlsnehamumbai
 
Russian Call Girl Brookfield - 7001305949 Escorts Service 50% Off with Cash O...
Russian Call Girl Brookfield - 7001305949 Escorts Service 50% Off with Cash O...Russian Call Girl Brookfield - 7001305949 Escorts Service 50% Off with Cash O...
Russian Call Girl Brookfield - 7001305949 Escorts Service 50% Off with Cash O...narwatsonia7
 
Bangalore Call Girls Marathahalli 📞 9907093804 High Profile Service 100% Safe
Bangalore Call Girls Marathahalli 📞 9907093804 High Profile Service 100% SafeBangalore Call Girls Marathahalli 📞 9907093804 High Profile Service 100% Safe
Bangalore Call Girls Marathahalli 📞 9907093804 High Profile Service 100% Safenarwatsonia7
 
Russian Call Girls in Bangalore Manisha 7001305949 Independent Escort Service...
Russian Call Girls in Bangalore Manisha 7001305949 Independent Escort Service...Russian Call Girls in Bangalore Manisha 7001305949 Independent Escort Service...
Russian Call Girls in Bangalore Manisha 7001305949 Independent Escort Service...narwatsonia7
 
Call Girl Service Bidadi - For 7001305949 Cheap & Best with original Photos
Call Girl Service Bidadi - For 7001305949 Cheap & Best with original PhotosCall Girl Service Bidadi - For 7001305949 Cheap & Best with original Photos
Call Girl Service Bidadi - For 7001305949 Cheap & Best with original Photosnarwatsonia7
 
Russian Call Girls Chennai Madhuri 9907093804 Independent Call Girls Service ...
Russian Call Girls Chennai Madhuri 9907093804 Independent Call Girls Service ...Russian Call Girls Chennai Madhuri 9907093804 Independent Call Girls Service ...
Russian Call Girls Chennai Madhuri 9907093804 Independent Call Girls Service ...Nehru place Escorts
 
Call Girls Chennai Megha 9907093804 Independent Call Girls Service Chennai
Call Girls Chennai Megha 9907093804 Independent Call Girls Service ChennaiCall Girls Chennai Megha 9907093804 Independent Call Girls Service Chennai
Call Girls Chennai Megha 9907093804 Independent Call Girls Service ChennaiNehru place Escorts
 
VIP Call Girls Tirunelveli Aaradhya 8250192130 Independent Escort Service Tir...
VIP Call Girls Tirunelveli Aaradhya 8250192130 Independent Escort Service Tir...VIP Call Girls Tirunelveli Aaradhya 8250192130 Independent Escort Service Tir...
VIP Call Girls Tirunelveli Aaradhya 8250192130 Independent Escort Service Tir...narwatsonia7
 
Call Girl Coimbatore Prisha☎️ 8250192130 Independent Escort Service Coimbatore
Call Girl Coimbatore Prisha☎️  8250192130 Independent Escort Service CoimbatoreCall Girl Coimbatore Prisha☎️  8250192130 Independent Escort Service Coimbatore
Call Girl Coimbatore Prisha☎️ 8250192130 Independent Escort Service Coimbatorenarwatsonia7
 
Call Girls Service Surat Samaira ❤️🍑 8250192130 👄 Independent Escort Service ...
Call Girls Service Surat Samaira ❤️🍑 8250192130 👄 Independent Escort Service ...Call Girls Service Surat Samaira ❤️🍑 8250192130 👄 Independent Escort Service ...
Call Girls Service Surat Samaira ❤️🍑 8250192130 👄 Independent Escort Service ...CALL GIRLS
 
Call Girls Service Pune Vaishnavi 9907093804 Short 1500 Night 6000 Best call ...
Call Girls Service Pune Vaishnavi 9907093804 Short 1500 Night 6000 Best call ...Call Girls Service Pune Vaishnavi 9907093804 Short 1500 Night 6000 Best call ...
Call Girls Service Pune Vaishnavi 9907093804 Short 1500 Night 6000 Best call ...Miss joya
 
Housewife Call Girls Hoskote | 7001305949 At Low Cost Cash Payment Booking
Housewife Call Girls Hoskote | 7001305949 At Low Cost Cash Payment BookingHousewife Call Girls Hoskote | 7001305949 At Low Cost Cash Payment Booking
Housewife Call Girls Hoskote | 7001305949 At Low Cost Cash Payment Bookingnarwatsonia7
 
Bangalore Call Girls Majestic 📞 9907093804 High Profile Service 100% Safe
Bangalore Call Girls Majestic 📞 9907093804 High Profile Service 100% SafeBangalore Call Girls Majestic 📞 9907093804 High Profile Service 100% Safe
Bangalore Call Girls Majestic 📞 9907093804 High Profile Service 100% Safenarwatsonia7
 
Call Girls Horamavu WhatsApp Number 7001035870 Meeting With Bangalore Escorts
Call Girls Horamavu WhatsApp Number 7001035870 Meeting With Bangalore EscortsCall Girls Horamavu WhatsApp Number 7001035870 Meeting With Bangalore Escorts
Call Girls Horamavu WhatsApp Number 7001035870 Meeting With Bangalore Escortsvidya singh
 
College Call Girls Pune Mira 9907093804 Short 1500 Night 6000 Best call girls...
College Call Girls Pune Mira 9907093804 Short 1500 Night 6000 Best call girls...College Call Girls Pune Mira 9907093804 Short 1500 Night 6000 Best call girls...
College Call Girls Pune Mira 9907093804 Short 1500 Night 6000 Best call girls...Miss joya
 
Call Girls Service in Bommanahalli - 7001305949 with real photos and phone nu...
Call Girls Service in Bommanahalli - 7001305949 with real photos and phone nu...Call Girls Service in Bommanahalli - 7001305949 with real photos and phone nu...
Call Girls Service in Bommanahalli - 7001305949 with real photos and phone nu...narwatsonia7
 
Call Girls Doddaballapur Road Just Call 7001305949 Top Class Call Girl Servic...
Call Girls Doddaballapur Road Just Call 7001305949 Top Class Call Girl Servic...Call Girls Doddaballapur Road Just Call 7001305949 Top Class Call Girl Servic...
Call Girls Doddaballapur Road Just Call 7001305949 Top Class Call Girl Servic...narwatsonia7
 

Recently uploaded (20)

Call Girls Whitefield Just Call 7001305949 Top Class Call Girl Service Available
Call Girls Whitefield Just Call 7001305949 Top Class Call Girl Service AvailableCall Girls Whitefield Just Call 7001305949 Top Class Call Girl Service Available
Call Girls Whitefield Just Call 7001305949 Top Class Call Girl Service Available
 
Call Girls In Andheri East Call 9920874524 Book Hot And Sexy Girls
Call Girls In Andheri East Call 9920874524 Book Hot And Sexy GirlsCall Girls In Andheri East Call 9920874524 Book Hot And Sexy Girls
Call Girls In Andheri East Call 9920874524 Book Hot And Sexy Girls
 
Russian Call Girl Brookfield - 7001305949 Escorts Service 50% Off with Cash O...
Russian Call Girl Brookfield - 7001305949 Escorts Service 50% Off with Cash O...Russian Call Girl Brookfield - 7001305949 Escorts Service 50% Off with Cash O...
Russian Call Girl Brookfield - 7001305949 Escorts Service 50% Off with Cash O...
 
Bangalore Call Girls Marathahalli 📞 9907093804 High Profile Service 100% Safe
Bangalore Call Girls Marathahalli 📞 9907093804 High Profile Service 100% SafeBangalore Call Girls Marathahalli 📞 9907093804 High Profile Service 100% Safe
Bangalore Call Girls Marathahalli 📞 9907093804 High Profile Service 100% Safe
 
Russian Call Girls in Bangalore Manisha 7001305949 Independent Escort Service...
Russian Call Girls in Bangalore Manisha 7001305949 Independent Escort Service...Russian Call Girls in Bangalore Manisha 7001305949 Independent Escort Service...
Russian Call Girls in Bangalore Manisha 7001305949 Independent Escort Service...
 
Call Girl Service Bidadi - For 7001305949 Cheap & Best with original Photos
Call Girl Service Bidadi - For 7001305949 Cheap & Best with original PhotosCall Girl Service Bidadi - For 7001305949 Cheap & Best with original Photos
Call Girl Service Bidadi - For 7001305949 Cheap & Best with original Photos
 
Russian Call Girls Chennai Madhuri 9907093804 Independent Call Girls Service ...
Russian Call Girls Chennai Madhuri 9907093804 Independent Call Girls Service ...Russian Call Girls Chennai Madhuri 9907093804 Independent Call Girls Service ...
Russian Call Girls Chennai Madhuri 9907093804 Independent Call Girls Service ...
 
Call Girls Chennai Megha 9907093804 Independent Call Girls Service Chennai
Call Girls Chennai Megha 9907093804 Independent Call Girls Service ChennaiCall Girls Chennai Megha 9907093804 Independent Call Girls Service Chennai
Call Girls Chennai Megha 9907093804 Independent Call Girls Service Chennai
 
Escort Service Call Girls In Sarita Vihar,, 99530°56974 Delhi NCR
Escort Service Call Girls In Sarita Vihar,, 99530°56974 Delhi NCREscort Service Call Girls In Sarita Vihar,, 99530°56974 Delhi NCR
Escort Service Call Girls In Sarita Vihar,, 99530°56974 Delhi NCR
 
VIP Call Girls Tirunelveli Aaradhya 8250192130 Independent Escort Service Tir...
VIP Call Girls Tirunelveli Aaradhya 8250192130 Independent Escort Service Tir...VIP Call Girls Tirunelveli Aaradhya 8250192130 Independent Escort Service Tir...
VIP Call Girls Tirunelveli Aaradhya 8250192130 Independent Escort Service Tir...
 
Call Girl Coimbatore Prisha☎️ 8250192130 Independent Escort Service Coimbatore
Call Girl Coimbatore Prisha☎️  8250192130 Independent Escort Service CoimbatoreCall Girl Coimbatore Prisha☎️  8250192130 Independent Escort Service Coimbatore
Call Girl Coimbatore Prisha☎️ 8250192130 Independent Escort Service Coimbatore
 
Call Girls Service Surat Samaira ❤️🍑 8250192130 👄 Independent Escort Service ...
Call Girls Service Surat Samaira ❤️🍑 8250192130 👄 Independent Escort Service ...Call Girls Service Surat Samaira ❤️🍑 8250192130 👄 Independent Escort Service ...
Call Girls Service Surat Samaira ❤️🍑 8250192130 👄 Independent Escort Service ...
 
Call Girls Service Pune Vaishnavi 9907093804 Short 1500 Night 6000 Best call ...
Call Girls Service Pune Vaishnavi 9907093804 Short 1500 Night 6000 Best call ...Call Girls Service Pune Vaishnavi 9907093804 Short 1500 Night 6000 Best call ...
Call Girls Service Pune Vaishnavi 9907093804 Short 1500 Night 6000 Best call ...
 
sauth delhi call girls in Bhajanpura 🔝 9953056974 🔝 escort Service
sauth delhi call girls in Bhajanpura 🔝 9953056974 🔝 escort Servicesauth delhi call girls in Bhajanpura 🔝 9953056974 🔝 escort Service
sauth delhi call girls in Bhajanpura 🔝 9953056974 🔝 escort Service
 
Housewife Call Girls Hoskote | 7001305949 At Low Cost Cash Payment Booking
Housewife Call Girls Hoskote | 7001305949 At Low Cost Cash Payment BookingHousewife Call Girls Hoskote | 7001305949 At Low Cost Cash Payment Booking
Housewife Call Girls Hoskote | 7001305949 At Low Cost Cash Payment Booking
 
Bangalore Call Girls Majestic 📞 9907093804 High Profile Service 100% Safe
Bangalore Call Girls Majestic 📞 9907093804 High Profile Service 100% SafeBangalore Call Girls Majestic 📞 9907093804 High Profile Service 100% Safe
Bangalore Call Girls Majestic 📞 9907093804 High Profile Service 100% Safe
 
Call Girls Horamavu WhatsApp Number 7001035870 Meeting With Bangalore Escorts
Call Girls Horamavu WhatsApp Number 7001035870 Meeting With Bangalore EscortsCall Girls Horamavu WhatsApp Number 7001035870 Meeting With Bangalore Escorts
Call Girls Horamavu WhatsApp Number 7001035870 Meeting With Bangalore Escorts
 
College Call Girls Pune Mira 9907093804 Short 1500 Night 6000 Best call girls...
College Call Girls Pune Mira 9907093804 Short 1500 Night 6000 Best call girls...College Call Girls Pune Mira 9907093804 Short 1500 Night 6000 Best call girls...
College Call Girls Pune Mira 9907093804 Short 1500 Night 6000 Best call girls...
 
Call Girls Service in Bommanahalli - 7001305949 with real photos and phone nu...
Call Girls Service in Bommanahalli - 7001305949 with real photos and phone nu...Call Girls Service in Bommanahalli - 7001305949 with real photos and phone nu...
Call Girls Service in Bommanahalli - 7001305949 with real photos and phone nu...
 
Call Girls Doddaballapur Road Just Call 7001305949 Top Class Call Girl Servic...
Call Girls Doddaballapur Road Just Call 7001305949 Top Class Call Girl Servic...Call Girls Doddaballapur Road Just Call 7001305949 Top Class Call Girl Servic...
Call Girls Doddaballapur Road Just Call 7001305949 Top Class Call Girl Servic...
 

Dermatopathology Generator

  • 1. Dermatopathology 1) Both mesenchymal and epithelial elements may be found in this tumor: A. Bednar tumor B. Cylindroma C. Chondroid syringomaCorrect Choice D. Microcystic adnexal carcinoma E. Folliculosebaceous cystic hamartoma Cutaneous mixed tumor, also known as Chondroid syringoma, represents an acquired hamartoma with folliculosebaceous-apocrine differentiation that has been generally interpreted as a form of adnexal adenoma (neoplasm). It has both a mesenchymal and epithelial component 2) What is the diagnosis? A. Papillary eccrine adenoma B. SyringomaCorrect Choice C. Adenoid cystic carcinoma D. Trichoadenoma E. Dermal duct tumor This is a syringoma, in which there are small cords and strands of epithelial cells, some in a “tadpole” configuration. There are scattered lumens, often lined with clear cells, with a bluish substance within them. No horn cysts are present 3) The diagnosis is: Image Link 1 Image Link 2 Image Link 3 Image Link 4 A. clear cell Bowen's disease B. extramammary Paget'sCorrect Choice C. epidermotropic sebaceous carcinoma D. epidermotropic Merkel cell carcinoma E. epidermotropic balloon-cell melanoma In extramammary Paget’s disease, there are epidermotropic, large cells with a bluish cytoplasm. At times these cells can form “nests”, but in general, there is usually a compressed basal layer beneath the “nests”. 4) This is a desmoplakin: A. Plakophilin B. Plakoglobin C. BPAg1 Correct Choice D. Beta-catenin 1
  • 2. E. Desmocollin Desmoplakins include desmoplakin 1, BPAg1, envoplakin, and periplakin 5) Histologically, adenoma sebaceum represent which of the following lesions? A. Angiokeratomas B. Angiofibromas Correct Choice C. Collagenomas D. Neurofibromas E. Smooth muscle hamartomas Adenoma sebaceum, fibrous papules and pearly penile papules all have similar features histologically, presenting as angiofibromas. Features include atrophic epidermis with patchy melanocytic hyperplasia and hyperkeratosis, vertically oriented collagen, increased fibroblasts and blood vessels 6) Lipomembranous change is seen in: A. Sclerosing panniculitis Correct Choice B. Sebaceous carcinoma C. Cystic sebaceous adenoma D. Hibernoma E. Mucocele Lipomembranous change is a non-specific histologic pattern that is most commonly seen in lipodermatosclerosis, which is also known as sclerosing panniculitis; this condition may be secondary to venous stasis 7) The diagnosis is: Image Link 1 Image Link 2 A. sarcoidosis B. foreign body C. lupus miliaris et disseminata D. lichen nitidusCorrect Choice E. lymphocytoma cutis 8) What is the diagnosis? A. Pilomatrixoma B. Metastatic carcinoma C. Tattoo D. Wood splinter E. Calcinosis cutisCorrect Choice This is calcinosis cutis, in which there is calcification (blue chunky material) in the dermis 2
  • 3. 9) The diagnosis is: A. Neurilemmoma B. Granular cell tumor C. Xanthoma Correct Choice D. Reticulohistiocytoma E. Neuroma NEEDS EXPLANATIONS 10) The diagnosis is: Image Link 1 Image Link 2 Image Link 3 Image Link 4 A. spiradenoma B. mastocytosis C. hidradenoma D. glomangiomaCorrect Choice E. Kimura's In glomangiomas, one sees multiple lumina lined by cells with pink cytoplasm and indisctinct borders with very round nuclei. Generally, the lumina are lined by one or two layers of glomus cells. 11) 62-year old female with history of acute myeloid leukemia presents with multiple edematous, erythematous papules after starting G-CSF. A. Bowel bypass dermatosis B. Leukocytoclastic vasculitis C. Erythema multiforme D. Polymorphous light eruption E. Sweet's syndromeCorrect Choice Sweet's syndrome, or acute febrile neutrophilic dermatoses, is often associated with AML and G- CSF. Histologically, there is marked dermal edema with a prominent infiltrate composed of neutrophils with leukocytoclasia. There is an absence of extensive vascular damage 12) Multiple such lesions are seen in this syndrome: Image Link 1 Image Link 2 Image Link 3 A. Nicolau and Balus' B. Alagille's C. Rubinstein-Taybi's D. Schopf's 3
  • 4. E. Brooke-SpieglerCorrect Choice In Brooke-Spiegler syndrome, there are multiple trichoepitheliomas, cylindromas, and spiradenomas. Nicolau and Balus’ syndrome has multiple eruptive syringomas, milia, and atrophoderma vermiculata. Alagille’s syndrome is the association of arteriohepatic dsyplasia with nevus comdonicus. Schopf’s syndrome associates multiple hidrocystomas with hypodontia, palmoplantar hyperkeratosis, and onychodystrophy. In Rubinstein-Taybi’s syndrome, patients are short of stature with broad thumbs and multiple pilomatricomas 13) This patient has multiple lesions showing the following histology. She has a family history of such lesions. You check her for: Image Link 1 Image Link 2 Image Link 3 A. fumarate hydrataseCorrect Choice B. transglutaminase C. epoxide hydrolase D. sulfatase E. urease 14) The diagnosis is:: Image Link 1 Image Link 2 Image Link 3 A. incontinentia pigmentiCorrect Choice B. pemphigus vulgaris C. epidermal nevus D. bullous pemphigoid E. dermatitis herpetiformis 15) The diagnosis is: Image Link 1 Image Link 2 Image Link 3 A. adenoid cystic carcinomaCorrect Choice B. trichoepithelioma C. dermal duct tumor D. syringoma E. tubular apocrine adenoma In adenoid cystic carcinoma, there are cords and tubules of basaloid cells, arranged in a cribiform pattern. The cystic spaces contain a bluish material. Perineural invasion is common. The cells stain positively for epithelial membrane antigen (EMA). 15) The diagnosis is: Image Link 1 4
  • 5. Image Link 2 Image Link 3 A. adenoid cystic carcinomaCorrect Choice B. trichoepithelioma C. dermal duct tumor D. syringoma E. tubular apocrine adenoma In adenoid cystic carcinoma, there are cords and tubules of basaloid cells, arranged in a cribiform pattern. The cystic spaces contain a bluish material. Perineural invasion is common. The cells stain positively for epithelial membrane antigen (EMA 16) For this patient, you request that the lab perform indirect immunofluorescence using what substrate? A. Monkey esophagus B. Mouse epithelium C. Rat bladder D. Guinea pig esophagusCorrect Choice E. Hep-2 cells This patient has pemphigus folicaceus, and indirect immunofluorescence works best on guinea pig esophagus 17) This patient may need blood tests to check her: A. Renal function B. White blood cell count C. Liver function D. ThyroidCorrect Choice E. Glucose Herpes gestationis is associated with an increased incidence of Graves’ disease. 18) The diagnosis is: A. Wegener’s granulomatosis B. Syphilis C. Fixed drug reaction D. Granuloma faciale Correct Choice E. Angiolymphoid hyperplasia with eosinophilia NEEDS EXPLANATIONS 19) Multiple clear cell acanthomas are associated with: 5
  • 6. A. Immunosuppression B. Ichthyosis Correct Choice C. Gastrointestinal polyps D. Breast cancer E. Cowden’s Clear cell acanthoma is associated with ichthyosis 20) The diagnosis is: Image Link 1 Image Link 2 Image Link 3 A. metastatic carcinoma B. Merkel cell carcinoma C. pyogenic granuloma D. bacillary angiomatous E. glomus tumorCorrect Choice In a glomus tumor, generally vascular spaces are not particularly prominent, and there are cords as well as solid areas of uniform cells with very monomorphous rounded nuclei 21) The diagnosis is: Image Link 1 Image Link 2 Image Link 3 Image Link 4 A. Large cell acanthoma B. Clear cell acanthomaCorrect Choice C. Hidroacanthoma simplex D. Poroma E. White sponge nevus In a clear cell acanthoma, there is regular acanthosis, often with some parakeratotic scale with neutrophils overlying it. The keratinocytes making up the thickened epidermis are often clear, or pale, due to an increased glycogen content secondary to a deficiency of phosphorylase. There is often a very abrupt demarcation at the margins of the acanthotic pale/clear cells and the normal epidermis 22) The gene defect: Image Link 1 Image Link 2 Image Link 3 A. DSG3 B. ATP2A2 C. ATP2C1Correct Choice D. SERCA2 6
  • 7. E. CHRNA9 In Hailey-Hailey disease, there is full-thickness acantholysis of the epidermis. The gene defect is in ATP2C1 23) This tumor is vimentin+ and cytokeratin+: A. Neurothekeoma B. Dermatofibroma C. Angiosarcoma D. Epithelioid sarcoma Correct Choice E. Nodular fasciitis Characteristic immunohistochemistry of epithelioid sarcoma is vimentin- and cytokeratin- positivity 24) The diagnosis is: Image Link 1 Image Link 2 Image Link 3 A. argyriaCorrect Choice B. minocycline-induced pigmentation C. hemochromatosis D. ochronosis E. gold effect In argyria, the silver deposits (black particles) are often seen around eccrine glands. 25) The diagnosis is: Image Link 1 Image Link 2 Image Link 3 A. traumatic neuroma B. acral angiofibroma C. accessory digit D. Koenen's tumor E. acquired digital fibrokeratomaCorrect Choice This is an acquired digital fibrokeratoma, with a thickened stratum corneum indicating an acral location and an absence of nerves in the dermis. Collagen is increased in the dermis 26) With immunofluorescence, the most likely pattern would be: Image Link 1 Image Link 2 Image Link 3 A. granular IgG B. granular IgACorrect Choice 7
  • 8. C. linear IgG, IgM, C3, and IgA D. tubular IgG E. linear IgM In dermatitis herpetiformis, there are characteristic clusters of neutrophils at the tips of dermal papillae. Immunofluorescence will show granular IgA in the dermal papillae. Neutrophils at the dermo-epidermal junction of a bulla can also be seen in bullous systemic lupus erythematosus, linear IgA disease, neutrophilic bullous pemphigoid, and inflammatory epidermolysis bullosa acquisita 27) The diagnosis is: Image Link 1 Image Link 2 Image Link 3 A. epidermal nevus B. poromaCorrect Choice C. bowenoid papulosis D. large cell acanthoma E. verruca In a poroma, there is a down-growing epidermal proliferation composed of uniform cells, often with indistinct cytoplasmic borders that have rounded, monomorphous nuclei. Within this proliferation, there are often ducts lined by an eosinophilic cuticle 28) Which immunohistochemical stain would be positive in eosinophilic granuloma? A. Congo red B. Mucin C. Cytokeratin 20 D. CD1aCorrect Choice E. HMB-45 Eosinophilic granuloma is a form of Langerhans Cell Histocytosis (LCH), previously called Histiocytosis X. Eosinophilic granuloma is a localized, benign form which is more common in males and generally affects the bones. All forms of LCH are characterized by the infiltration of Langerhans cells on pathology, which staing for S-100, CD1a and contain cytoplasmic birbeck granules 29) What is the diagnosis? A. Lichen nitidusCorrect Choice B. Lupus C. Lichen planus D. Lichen planus-like keratosis E. Lichen striatus The histologic features shown are indicative of lichen nitidus. Lichen nitidus has a very characteristic "claw clutching ball" appearance where the rete ridges extend downward around a lichenoid infiltrate 8
  • 9. 30) An increased number of miniaturized hairs are seen in: A. Lichen planopilaris and alopecia areata B. Alopecia areata Correct Choice C. Trichotillomania D. Lichen planopilaris E. Telogen effluvium In alopecia areata, especially early stages, increased miniaturized catagen hairs can be seen in addition to the finding of peribulbar lymphocytes resembling a “swarm of bees.” Lichen planopilaris is a scarring alopecia in which vertical tracts of fibrosis are seen in place of follicles. Trichotillomania displays follicular plugging, trichomalacia, pigmented casts, hemorrhage, and increased catagen hairs on biopsy. Telogen effluvium is characterized by an increased number of telogen hairs 31) The diagnosis is: Image Link 1 Image Link 2 Image Link 3 A. Monsel's reaction B. chrysiasis C. amalgam tattoo D. minocycline-induced pigmentation E. ochronosisCorrect Choice In ochronosis, there is slightly thickened banana-shaped collagen in the superficial dermis that appears yellow-brown in color 32) The diagnosis of this biopsy is: A. Erythema nodosum B. Polyarteritis nodosaCorrect Choice C. Granuloma annulare D. Leukocytoclstic vasculitis E. Reumatoid nodule Polyarteritis nodosa is a necrotizing vasculitis of medium-sized arteries in the derma-subcutaneous junction. It is a septal panniculitis as a result of vasculitis. There is more fibrinous changes than in thrombophlebitis and less necrosis than erythema induratum 33) What is the pathologic diagnosis of this lesion? A. Acanthosis nigricans B. Verruca planaCorrect Choice C. Seborrheic keratosis D. Normal skin E. Epidermal nevus 9
  • 10. The histologic characteristics of verruca plana are presence of hyperkeratosis and acanthosis. Koilocytes appear in the upper one-third of the epidermis 34) Similar follicles can be seen on skin from the: Image Link 1 Image Link 2 A. fiinger B. glans penis C. nose D. labia majora E. eyelidCorrect Choice Vellus hair follicles are seen commonly in accessory tragic and eyelid skin 35) The diagnosis is: Image Link 1 Image Link 2 Image Link 3 Image Link 4 A. Bowen's disease B. melanoma C. sebaceous carcinoma D. pagetoid reticulosis E. Paget's diseaseCorrect Choice In Paget’s disease, there are epidermotropic cells in a variably acanthotic epidermis. The epidermotropic cells have abundant cytoplasm that is often blue-gray in color. The cells are single or in nests throughout the epidermis. There should be a compressed rim of basal cells beneath nests that are located near the dermoepidermal junction 36) The diagnosis is:: Image Link 1 Image Link 2 Image Link 3 A. mixed tumor B. follicular mucinosisCorrect Choice C. alopecia areata D. lichen planopilaris E. pityrosporum folliculitis 37) Eosinophils are typically found in each of the following except: A. Incontinentia pigmenti B. PlasmacytomaCorrect Choice C. Urticaria 10
  • 11. D. Lichenoid drug reactions E. Pemphigus vulgaris Plasmacytomas are characterized by the presence of monoclonal plasma cells. Plasmacytomas may occur from primary cutaneous focus or secondarily from myeloma 38) The diagnosis is: Image Link 1 Image Link 2 Image Link 3 Image Link 4 Image Link 5 A. Merkel cell carcinomaCorrect Choice B. neuroblastoma C. plasmacytoma D. Lymphoma E. melanoma This is a Merkel cell carcinoma, in which there is a dense collection of small blue cells with scant cytoplasm in the dermis. The cells are sometimes arranged in trabeculae and other times in nodules. The cells appear very blue on low power, and on higher power have somewhat pale nuclei that have paler/darker foci within them in a salt-and-pepper pattern 39) Which of the following lesions demonstrates a pseudo-Darier’s sign? A. Mastocytoma B. Spitz nevus C. Smooth muscle hamartomaCorrect Choice D. Pilomatricoma E. Bullous pemphigoid Smooth muscle hamartomas are benign tumors which arise from smooth muscle of the dermis. Pseudo-Darier's sign may be elicited due to transient piloerection after rubbing. Histologically, red- orange bundles and fascicles are present with blunt-ended nuclei 40) The diagnosis is: Image Link 1 Image Link 2 Image Link 3 A. apocrine adenoma B. amyloidosis C. Masson's tumorCorrect Choice D. intravascular pyogenic granuloma E. papillary digital adenocarcinoma 41) The most likely diagnosis for this painful neoplasm is: 11
  • 12. A. Glomus tumor B. Neurilemmoma C. Cutaneous endometriosisCorrect Choice D. Blue rubber bleb nevus E. Angiolipoma Cutaneous endometriosis usually occurs after gynecologic surgery. The rich, cellular stroma with regularly shaped channels and glandular structures are typical. Hemorrhage may also be present in deeper sections 42) What infectious agent is most likely responsible for this reaction of fibrin and antibodies which help to prevent phagocytosis? A. ActinomycosisCorrect Choice B. Anthrax C. Candida albicans D. Ricketsii species E. Nocardia Hoeppli-Splendore reaction is characterized histologically by intensely eosinophilic material consisting of fibrin and antibodies. Causes of the phenomenon include Actinomycosis israelii, Staph aureus, Proteus, Pseudomonas and E. coli 43) Goblet cells are seen in: A. Bronchogenic cyst Correct Choice B. Steatocystoma C. Cutaneous ciliated cyst D. Dermoid cyst E. Endometriosis Bronchogenic cysts have a pseudostratified cuboidal or columnar lining that is ciliated; goblet cells are found in the lining as well 44) The diagnosis is: A. Metastatic carcinoma B. Mixed tumor Correct Choice C. Papillary eccrine adenoma D. Nodular hidradenoma E. Mucinous carcinoma NEEDS EXPLANATIONS 45) The most common cause of this in the U.S. is: Image Link 1 Image Link 2 Image Link 3 12
  • 13. A. Microsporum distortum B. Epidermophyton floccosum C. Trichophyton rubrumCorrect Choice D. Microsporum canis E. Trichophyton mentagrophytes Trichophyton rubrum is the most common cause of Majocchi’s granuloma, a type of folliculitis where the dermatophyte likely tracks down the follicle and creates a foreign-body-type inflammatory reaction in the dermis 46) This patient recently developed this rash. You decide to patch test her, but in the meanwhile you tell her to avoid: A. Primin B. Abietic acid C. Benzocaine D. CinnamonCorrect Choice E. Chamomile This patient likely has a fragrance allergy. Patients allergic to fragrances also need to avoid certain spices like cinnamon 47) Multiple such lesions may be seen associated with: Image Link 1 Image Link 2 Image Link 3 A. Cowden's B. Wermer'sCorrect Choice C. Cowper's D. Werner's E. Brooke's MEN type I (Wermer's Syndrome) is sometmes associated with multiple angiofibromas. Tuberous sclerosis is also associated with adenoma sebaceum (angiofibromas 48) Turk cells are found in what infection? A. Rubeola B. Mumps C. Roseola D. Syphilis E. RubellaCorrect Choice Turk cells are atypical lymphocytes found in rubella 49) The diagnosis is: Image Link 1 13
  • 14. Image Link 2 Image Link 3 A. Masson's B. bacillary angiomatosis C. hemangioendothelioma D. fibrosing pyogenic granulomaCorrect Choice E. apocrine adenoma 50) Which disease process best describes Texier's disease? A. Deposition disorder B. PanniculitisCorrect Choice C. Granulomatous disease D. Neutrophilic dermatosis E. Infectious process Texier's disease is a panniculitis secondary to vitamin K injections causing sclerotic lesions with lilac borders on the buttocks and thighs resembling a cowboy belt and holster 51) A patient with dystrophic nails and multiple lesions with this histologic finding may have what gene defect? A. Phosphorylase B. Keratin 6b/17Correct Choice C. Beta-catenin D. Keratin 6a/16 E. Keratin 1/10 Pachonychia congenita type II (also known as Jackson-Lawler) is a autosomal dominant disorder characterized by natal teeth, steatocystoma multiplex, and pincer nails. Steatocystomas have cyst walls that are intricately folded or crenulated. Sebaceous glands are present within the cyst wall 52) What is the diagnosis? A. Warty dyskeratoma B. Molluscum contagiosumCorrect Choice C. Trichilemmoma D. Ecthyma contagiosum E. Orf This is a lesion of Molluscum contagiosum, in which there is a lobulated down-growth (cup-shaped) of keratinocytes; centrally towards the surface, the keratinocytes are enlarged with cytoplasmic pink inclusions (Henderson-Patterson bodies 53) Multiple such lesions can be associated with: Image Link 1 14
  • 15. Image Link 2 Image Link 3 A. Gaucher's B. Sipple's C. fucosidosisCorrect Choice D. Louis-Barr E. Bourneville's multiple angiokeratomas in a bathing trunk distribution may be associated with Fabry's disease (angiokeratoma corporis diffusum) or fucosidosis as well as some other storage diseases as well as possibly being a normal finding. There has been a recent report of angiokeratomas in a bathing trunk distribution in a woman with no other signs of a storage disorder 54) What is the diagnosis? A. Lipoid protinosis B. Erythrpoietic protoporphyria C. Lichen sclerosus et atrophicusCorrect Choice D. Lichen amyolidosus E. Morphea This is lichen sclerosus et atrophicus in which there is hyperkeratosis overlying an atrophic epidermis. Underlying the epidermis is a layer of homogenized light pink collagen. Beneath that, there is a somewhat band-like inflammatory infiltrate of predominantly lymphocytes, but also histiocytes and plasma cells 55) What is the diagnosis? A. Seborrheic keratosis B. Hidroacanthoma simplex C. Epidermal nevus D. Fibroepithelioma of PinkusCorrect Choice E. Nevus sebaceus Fibroepithelioma of Pinkus is a form of basal cell carcinoma. Histology shows long, thin, anastomosing strans of basal cell embedded in fibrous stroma with many connections to the epidermis 56) A newborn infant presents with bullous lesions. Based upon the pathology, what is the most likely gene defect causing her skin condition? A. Keratin 1 and 10 B. PAX3 C. NEMOCorrect Choice D. Keratin 5 and 14 E. SPINK5 15
  • 16. Incontinentia pigmenti is an X_linked dominant genodermatosis which presents in the newborn period with vesicles in a Blaschkoid distribution. A biopsy would show eosinophilic spongiosis with dyskeratotic keratinocytes and pigment incontinence. A gene defect in NEMO has been identified as the cause for the constellation of fingings 57) The most common location for this lesion would be: A. Upper lip B. Sole of footCorrect Choice C. Buttock D. Finger E. Groin Eccrine poroma is a benign, solitary tumor arsing in the lower portion of the epidermis. It is characterized by small, uniformly cuboidal cells with deeply basophilic nuclei. The tumor mass is assembled in broad anastomosing bands and may have narrow ductal lumina lined by eosinophilic cuticle 58) The diagnosis is: Image Link 1 Image Link 2 Image Link 3 A. microcystic adenexal carcinoma B. breast carcinomaCorrect Choice C. neuroendocrine carcinoma D. tubular apocrine adenoma E. infiltrative basal cell carcinoma The diagnosis is metastatic breast carcinoma. In this example, there is a dense collection of cells throughout the ermis. On closer examination, tehre are strands/cords of cells as well as some cells arranged around lumina infiltrating through the dermis 59) Steatocystoma multiplex is associated with: A. Jackson-Lawler Correct Choice B. Jadassohn-Lewandowsky C. Zinsser-Engman-Cole D. Schaufer-Brunauer E. Touraine-Solente-Gole Jackson-Lawler (Jackson-Sertoli) is known as pachyonychia congenital type 2. Multiple steatocysts can be seen in this condition 60) Weibel-Palade bodies are seen in: A. Spitz Nevi B. Endothelial cells Correct Choice C. Cells infected with MCV 16
  • 17. D. Plasmacytoid Cells E. Malakoplakia Weibel-Palade bodies are seen in endothelial cells and are therefore found in vascular lesions. Kamino bodies are found in Spitz nevi. Henderson Patterson bodies are seen in molluscum. Dutcher bodies are intranuclear inclusions seen in plasmacytoid cells. Michaelis Gutmann bodies are partially digested bacteria seen in malakoplakia 61) Eosinophilia-Myalgia syndrome is caused by: A. Norwegian salt-petter B. Unadultered Spanish grapeseed oil C. Excessive anaerobic exercise D. L-Tryptophan Correct Choice E. Pb intoxication The eosinophilia myalgia syndrome is characterized by marked peripheral eosinophilia with a clinical spectrum of signs and symptoms, including generalized myalgias, pneumonitis, myocarditis, neuropathy, encephalopathy and fibrosis. Many patients progress to a clinical picture clinically indistinguishable from eosinophilic fasciitis. The disease is caused by the ingestion of certain lots of L-tryptophan 62) Blue-gray pigmentation on the legs secondary to minocycline on biopsy stains with: A. Fontana Masson and Perls Correct Choice B. All of these answers are correct C. Sudan black D. Fontana Masson E. Perls There are three types of pigmentary change that are caused by minocycline. The blue-gray pigmentation on the legs and the blue pigment in scars is thought to be secondary to a drug-protein complex deposited in the dermis. The blue-gray pigment on the legs stains with Perls and Fontana- Masson. The blue in scars (often on the face) stains with Perls. The muddy-brown discoloration on sun-exposed areas shows increased basilar pigment and melanin incontinence on biopsy. It is likely secondary to phototoxicity. 63) The diagnosis is: Image Link 1 Image Link 2 Image Link 3 A. Artecoll reaction B. goutCorrect Choice C. mucinous carcinoma D. Urbach-Wiethe's E. Hunter's In gout, there are characteristic amorphous light pink masses of material within which it is sometimes possible to see outlines of needle-like spaces. The urate crystals can only be seen if 17
  • 18. alcohol fixation is used. The amorphous material is generally surrounded by histiocytes and foreign- body giant cells 64) The histologic finding of "shoulder parakaratosis", parakeratosis with prediliection for the follicular ostia, is characteristic of pityriasis rubra pilaris as well as: A. Stasis dermatitis B. Atopic dermatitis C. Seborrheic dermatitisCorrect Choice D. Nummular dermatitis E. Allergic contact dermatitis Parakeratosis refers to pyknotic keratinocyte nuclei in the stratum corneum, where nuclei are not usually present. It is common in diseases with changes in the epidermis. Histologically seborrheic dermatitis can shows "shoulder parakeratosis" with epidermal spongiosis. Histologically atopic, nummular and contact dermatitis present with spongiosis with or without vesicles. Stasis dermatitis presents with more dilated papillary dermal small blood vessels and hemosiderin 65) The diagnosis is: Image Link 1 Image Link 2 Image Link 3 Image Link 4 Image Link 5 Image Link 6 A. scleroderma, early B. erythema induratum C. sarcoidosis D. erythema nodosumCorrect Choice E. subcutaneous granuloma annulare In erythema nodoosum, one sees a predominantly septal panniculitis with some septal thickening and fibrosis and an inflammatory infiltrate within the septae composed of lymphocytes, histiocytes, eosinophils, and giant cells 66) The diagnosis is: A. Psoriasis B. Bowen's disease C. Clear cell acanthoma Correct Choice D. Trichilemmoma E. Poroma NEEDS EXPLANATIONS 67) Which type of artifact is shown here? A. Electrodessication 18
  • 19. B. Dessication C. Gel foamCorrect Choice D. Microtome knife chatter E. Freeze The presence of deeply basophilic, wavy, angulated foreign material characteristic of gel foam artifact 68) This is associated with MEN IIa: A. Malignant peripheral nerve sheath tumor B. Macular amyloidosis Correct Choice C. Mucocele D. Neurothekeoma E. Chondroid syringoma Macular amyloidosis is associated with MEN IIa 69) The diagnosis is: A. Lymphoma B. Small cell melanoma C. Glomus tumor D. Merkel cell carcinoma Correct Choice E. Rhabdomyosarcoma NEEDS EXPLANATIONS 70) The diagnosis is: Image Link 1 Image Link 2 Image Link 3 A. acrospiroma B. reticulated seborrheic keratosis C. fibroepithelioma of PinkusCorrect Choice D. syringofibroadenoma E. tumor of the follicular infundibulum In fibroepithelioma of Pinkus, there is a reticulated network of basaloid cells coming off of the epidermis in a plate-like fashion. There is some peripheral palisading of cells. The cells are embedded in a fibrotic stroma 71) Paraproteinemia is associated with all except: A. Plane xanthoma B. Necrobiotic xanthogranuloma 19
  • 20. C. Scleromyxedema D. Sclerosing panniculitis Correct Choice E. Scleredema Sclerosing panniculitis (lipodermatosclerosis) displays characteristic changes in the fat (lipomembranous change); it is not associated with paraproteinemia. Generalized plane xanthomas, scleromyxedema, necrobiotic xanthogranuloma, scleredema, erythema elevatum diutinum, xanthoma disseminatum, and pyoderma gangrenosum have all been associated with a paraproteinemia 72) The diagnosis is: A. Cylindroma B. Trichoblastoma C. Hidradenoma D. Acrospiroma E. Spiradenoma Correct Choice NEEDS EXPLANATIONS 73) All of the following are true of reticulohistiocytoma except: A. Trauma is precipitating factor B. Rare occurrence in children C. Association with arthritisCorrect Choice D. Immunostaining is positive for OKM1 E. Giant cells with “ground-glass” cytoplasm Reticulohistiocytomas, also called giant cell reticulohistiocytomas, occur almost exclusively in adults. They are generally solitary, and unlike the multicentric type, are not associated with mutilating arthritis or predisposition for malignancy 74) What is the diagnosis? A. Spiradenoma B. CylindromaCorrect Choice C. Dermal duct tumor D. Syringocystadenoma papilliferum E. Trichoblastoma This is a cylindroma, in which there is a jigsaw puzzle type arrangement of islands of basaloid cells with intervening fibrous, pink stroma. The basaloid cells are sometimes rimmed by a thick, pink basement membrane 75) Mulberry cells contain increased: A. Mitochondria Correct Choice B. Phagolysosomes and mitochondria 20
  • 21. C. Phagolysosomes D. Golgi E. Ribosomes Hibernomas commonly arise in the neck, axillae, and posterior shoulder. The cells are multivacuolated and resemble mulberries; the cells are filled with mitochondria, as are the cells in normal brown fat 76) The endemic form of this disease may be transmited by: Image Link 1 Image Link 2 Image Link 3 A. Glossina B. SimuliumCorrect Choice C. Lutzomyia D. Phlebotomus E. Triatoma Triatoma (reduviid bug) species transmit American trypanosomiasis. Glossina is the genus of tsetse flies that transmit African trypanosomiasis. Simulium is the genus of the black fly that can tranmit Onchocerciasis and possibly the endemic form of pemphigus foliceus (fogo selvagem). Phlebotomus and Lutzomyia are types of sandflies that can transmit Leishmaniasis, Carrion’s disease, and viral sandfly fever 77) Which of the following hitologic features would be most helpful in differentiating lichenoid drug eruption from lichen planus? A. Squamatization of the basal layer B. Band-like infiltrate with “Saw-tooth” rete ridges C. Parakeratosis and eosinophilsCorrect Choice D. Presence of pruritus E. Civatte bodies Lichenoid drug eruptions share clinical and histopathologic features with lichen planus. Sometimes differentiation is not possible; however, eosinophil, parakeratosis, and a deeper perivascular infiltrate is more suggestive of lichenoid drug. Implicated medications include captopril, penicillamine, and chloroquine 78) In this patient, this test will be helpful in making the diagnosis: A. Direct immunofluorescenceCorrect Choice B. Tissue culture C. Fluorescent antibody test for herpes D. Patch test E. KOH exam This patient has penicillamine-induced pemphigus foliaceus. Direct immunofluorescence testing will be very helpful as it should reveal intercellular antibodies within the epidermis 21
  • 22. 79) This patient says the rash is spreading and not controlled with topical therapy. You give him a course of oral treatment that lasts: A. 1 week B. 3 weeksCorrect Choice C. 5 weeks D. 4 weeks E. 2 weeks Generally, for poison ivy dermatitis, if patients are given a course of oral steroids, the course should be at least 3 weeks long, as if the duration is shorter, patients may develop a rapid rebound 80) Langerhans cells express or are characterized by all of the following except: A. HLA-DR B. ChromagraninCorrect Choice C. Birbeck granules D. CD1a E. S-100 Chromagranin stain neuroendocrine cells, Merkel cellcarcinomas and eccrine glands. They do not stain Langerhans cells 81) What is the diagnosis? A. Dermatomyofibroma B. Leiomyosarcoma C. Palisaded encapsulated neuromaCorrect Choice D. Leiomyoma E. Traumatic neuroma This is a palisaded encapsulated neuroma, in which there are small bundles of cells that have wavy, thin (elongated) nuclei and pink cytoplasm. The cells are separated by artifactual clefting. The bundles are often located very superficially. Encapsulation is often incomplete/not obvious 82) This woman should have a workup for: A. Nephrolithiasis B. HemochromatosisCorrect Choice C. Lymphoma D. Thalassemia E. Pancreatic cancer Porphyria cutanea tarda has been shown to be associated with hemochromatosis. Patients with porphyria cutanea tarda have mutations in the HFE gene, and early detection of mutations can improve life expectancy for these patients 22
  • 23. 83) The diagnosis is: Image Link 1 Image Link 2 Image Link 3 A. myxedema B. digital mucous cystCorrect Choice C. reticulated erythematous mucinosis D. mucinous granuloma annulare E. papular mucinosis In a digital mucous cyst, one sees a collection of mucin in the dermis beneath acral skin. This entity is not a true cyst as there is no epithelial lining to the cyst. The mucin is largely composed of hyaluronic acid 84) The diagnosis is: Image Link 1 Image Link 2 Image Link 3 Image Link 4 Image Link 5 A. neurothkeoma B. palisaded encapsualted neuromaCorrect Choice C. amelanotic blue nevus D. dermatofibroma E. leiomyoma In palisaded encapsulated neuroma, there are broad fascicles of spindle cells set in a clear matrix. The fascicles of spindle cells are sometimes clearly separated from the surrounding normal dermis by a capsule, but other times blend into the dermis. The spindle cells have elongated, thin/tapered nuclei. Palisading of nuclei is often not obvious 85) The diagnosis is: Image Link 1 Image Link 2 Image Link 3 A. granuloma faciale B. pigmented purpura C. leukocytoclastic vasculitisCorrect Choice D. mastocytosis E. acrodermatitis of Mali In leukocytoclastic vasculitis, on low power there is an inflammatory infiltrate generally clustered around the vessels (although sometimes more dense and interstitial). There is extravasation of erythrocytes around vessels with predominantly neutrophils around vessels and often within the walls of the vessels. There is fragmentation of neutrophilic nuclei (“nuclear dust”) with fibrin (pink amorphous material) within the walls of vessels and sometimes frank destruction of vessels 86) Which type of artifact is illustrated here? 23
  • 24. A. ElectrodessicationCorrect Choice B. Gel foam C. Knife chatter D. Dessication E. Freeze The elogation of cells and spindling of nuclei with typical "string bean" appearance are characteristic of electrodessication artifact 87) This patient’s biopsy will likely show: A. Mononuclear cells with abundant cytoplasm around superficial vessels B. Eosinophils at the dermoepidermal junction C. Leukocytoclasia around superficial vessels D. Lymphocytes at the dermoepidermal junctionCorrect Choice E. Neutrophils at the dermoepidermal junction This patient has erythema multiforme, and biopsy should show a lichenoid infiltrate of lymphocytes at the dermoepidermal junction 88) The promontory sign is seen in: A. Tufted angioma B. Acroangiodermatitis of Mali C. Kaposi’s sarcoma Correct Choice D. Glomeruloid hemangioma E. Spindle cell hemangioendothelioma The promontory sign refers to the formation of new vessels around existing vessels and adnexal structures. This is seen in Kaposi’s 89) What is the diagnosis? A. Lupus erythematosus B. Mycosis fungoides C. Poroma D. Porokeratosis E. PsoriasisCorrect Choice This is psoriasis, in which there is parakeratosis with entrapped neutrophils overlying a regularly acanthotic epidermis. There is hypogranulosis of the epidermis with increased mitoses in the basal layer. There are thinned suprapapillary plates with dilated vessels in the superficial dermal papillae. There is a lympho-histiocytic infiltrate around superficial vessels. Occasionally, clusters of neutrophils can be seen in the stratum spinosum (spongiform pustules of Kogoj 90) What is the diagnosis? A. Melanoma 24
  • 25. B. Extramammary Paget’sCorrect Choice C. Sebaceous carcinoma D. Bowen’s E. Condyloma This is an example of extramammary Paget’s, in which there are atypical cells singly and in groups within the epidermis. The cells have abundant bluish cytoplasm 91) This is secondary to: Image Link 1 Image Link 2 A. paraproteinemia B. a dull blade C. silicone injections D. gel foamCorrect Choice E. metastatic carcinoma Gel foam in tissue sections is a characteristic wavy material that stains bluish-gray 92) The predominant location of the cleft in transient neonatal pustular melanosis is: A. Suprabasal B. Basement membrane zone C. Subcorneal/granularCorrect Choice D. Dermal E. Basal keratinocytes Transient neonatal pustular melanosis is an idiopathic pustular eruption of newborns, mostly on the chest, that heals with hyperpigmentation. It is most common on pigmented individuals. Histologically it presents as subcorneal pustules with eosinophils and neutrophils 93) What is the diagnosis? A. ChondrodermatitisCorrect Choice B. Granular cell tumor C. Bromoderma D. Lichen simplex chronicus E. Actinic keratosis This is chondrodermatitis nodularis helices, in which there is hyperkeratosis and parakeratosis overlying an altered/thickened epidermis. Beneath that area, in the dermis, there is often fibrosis. Flanking the fibrosis on either side, there is a proliferation of vessels and inflammation (resembling granulation tissue 94) A lichenoid infiltrate that surrounds eccrine glands is seen in: A. Lichen planus 25
  • 26. B. Lichenoid purpura C. Lichen striatusCorrect Choice D. Lichenoid drug rection E. Lichen planopilaris Lichen striatus is an uncommon inflammatory dermatitis seen most commonly in children aged 5 to 15. It presents unilaterally along Blaschko's lines as raised, slightly scaly, erythematous papules, which are often pruritic. These lesions typically regress spontaneously within a year. The histopathologic features of lichen striatus include a superficial perivascular inflammatory lymphohistiocytic infiltrate with rare plasma cells and eosinophils. There is a focal lichenoid infiltrate in the papillary dermis with basilar vacuolar alteration and necrotic keratinocytes. Spongiosis with exocytosis of lymphocytes can be seen in the epidermis. A specific and distinctive feature of lichen striatus is the presence of an inflammatory infiltrate that surrounds hair follicles and eccrine glands 95) Similar follicles can be seen on skin from the: A. EyelidCorrect Choice B. Nose C. Finger D. Glans penis E. Labia majora Vellus hair follicles are seen commonly in accessory tragic and eyelid skin 96) What is the diagnosis? A. Incontinentia pigmenti B. Lichen striatus C. Lichen simplex chronicus D. Lichen planusCorrect Choice E. Pityriasis lichenoides et varioliformis acuta This is lichen planus, in which there is hyperkeratosis (and no parakeratosis), irregular acanthosis of the epidermis, hypergranulosis (often in wedge shapes), saw-toothing of the basal layer, and a band-like inflammatory infiltrate of predominantly lymphocytes (usually no eosinophils) at the dermoepidermal junction. Occasionally, artifactual clefting can be seen at the dermoepidermal junction (Max-Joseph space). Colloid bodies/Civatte bodies (amorphous pink material in globs) may also be seen near the dermoepidermal junction 97) This reaction is most likely secondary to: Image Link 1 Image Link 2 A. nicotinamide B. tetracycline C. mycophenolic acid D. captoprilCorrect Choice E. cyclosporine 26
  • 27. Bullous pemphigoid can be drug-induced and a common inciting drug is captopril. Other causes include lasix, nalidixic acid, penicillamine, antibiotics (penicillin, amoxicillin, ampicillin), and PUVA 98) All have been associated with increased risk of breast cancer except: A. Birt-Hogg-Dube Correct Choice B. ataxia telangiectasia C. Multicentric reticulohistiocytosis D. Peutz-Jeghers E. Cowden’s Birt-Hogg-Dube is associated with renal cancer and thyroid cancer. Female carriers of a mutated ATM (homozygous mutations ATM cause ataxia telangiectasia) have an increased risk of breast cancer 99) The diagnosis is: Image Link 1 Image Link 2 Image Link 3 A. microcystic adnexal carcinoma B. mixed tumorCorrect Choice C. syringofibroadenoma D. mucinous carcinoma E. papillary eccrine adenoma Some authors separate mixed tumors into eccrine and apocrine types. In the eccrine mixed tumor (pictured here), there are cords, clusters, and strands of basaloid cells forming lumina, some lined by eosinophilic cuticles. These clusters of cells are embedded in a bluish myxoid/cartilaginous stroma 100) The predominant location of the cleft in acropustulosis of infancy is: A. Dermal B. Basment mebrane zone C. Subcorneal/granularCorrect Choice D. Suprabasal E. Basal keratinocytes Acropustulosis of infancy presents as idiopathic pustules on acral skin. Diagnosis is made only after other causes of pustules have been ruled out, and it usually resolves in a few years. The cleft in acropustulosis of infancy is subcorneal/granular with neutrophils 101) Cicatricial pemphigoid antibodies directed against this are associated with high frequency of malignancy: A. Laminin 5 Correct Choice B. Beta4-integrin C. BPAg2 27
  • 28. D. Laminin 6 E. All of these answers are correct Anti-laminin 5 cicatricial pemphigoid (CP) is also known as anti-epiligrin CP. Anti-epiligrin CP is associated with an increased frequency of internal adenocarcinomas. Laminin 5 is composed of three chains (heterotrimer), alpha3, beta3, gamma2. Antibodies are frequently directed against the alpha3 chain, and so cross-reactivity can be observed with laminin 6, as laminin 6 (alpha3beta1gamma1) has the alpha3 chain as well. Beta4-integrin antibodies have been associated with ocular CP. BPAg2 antibodies are seen in CP patients that have mucosal as well as skin disease 102) This patient may have antibodies to: Image Link 1 Image Link 2 Image Link 3 A. metalloproteinase B. transglutaminase C. myeloperoxidaseCorrect Choice D. proteinase-3 E. aminotransferase The figures are consitent with polyarteritis nodosa. Patients with polyarteritis nodosa may have antibodies to p-ANCA or myeloperoxidase 103) The diagnosis is: Image Link 1 Image Link 2 Image Link 3 Image Link 4 A. malignant fibrous histiocytoma B. nodular fascitisCorrect Choice C. epithelioid sarcoma D. neurofibroma E. dermatofibrosarcoma protuberans In nodular fasciitis, there is a ill-defined deep (often extending into fat) proliferation of plump spindle cells that on higher power resemble “tissue-culture fibroblasts” with elongated cytoplasm often set in a background of many small vessels and extravasated erythrocytes. The spindle cells are arranged haphazardly, and the stroma is often myxoid. Mitoses are common 104) What is the diagnosis? A. Psoriasis B. Granular parakeratosisCorrect Choice C. Lichen nitidus D. Dermatophyte E. Lichen planus 28
  • 29. Granular parakeratosis results from abnormal keratinization which generally occurs in the flexural areas. Histologically, the thick parakeratotic layer with retention of keratohyaline granules. In addition, the granular layer is preserved with relatively normal epidermis 105) What is the diagnosis? A. Mastocytosis B. Leprosy C. Sarcoid D. Lichenoid actinic keratosis E. Lichen nitidusCorrect Choice This is lichen nitidus, in which there is a “ball” of lymphocytes and histiocytes in the superficial dermis abutting the epidermis surrounded on both sides by “claws” of the epidermis (rete 106) Which of the following drugs has been known to cause pyogenic granuloma? A. CapecitabineCorrect Choice B. Paclitaxel C. Isosfamide D. Daunorubicin E. Mithramycin Systemic retinoids, indinavir and capecitabine have all been describe to cause pyogenic granulomas 107) Which of the following histologic features is seen in aging skin? A. Increased sebum production B. Increased number of terminal hairs C. Thickened dermal-epidermal junction D. Fewer Langerhans cellsCorrect Choice E. Increased mast cells Histologic features of aging epidermis include flattened dermo-epidermal junction, occasional nuclear atypia, decrease in the number of melanocytes and Langerhans cells. Changes that are present in the dermis include atrophy, decrease in fibroblasts, mast cells and blood vessels 108) In this patient, this test will be helpful in making the diagnosis: A. Patch test B. Fluorescent antibody test for herpesCorrect Choice C. Tissue culture D. KOH exam E. Indirect immunofluorescence This patient has herpes zoster. A direct fluorescent antibody test for the varicella zoster virus can be performed to confirm the diagnosis. 29
  • 30. 109) This patient also has anemia. He needs screening for:: Image Link 1 Image Link 2 Image Link 3 A. malignancyCorrect Choice B. liver cirrhosis C. diabetes D. pulmonary firbrosis E. immunosuppression The histology shows numerous neutrophils in the dermis with a lack of vasculitis, consistent with Sweet’s syndrome. In a patient with Sweet’s syndrome, the presence of anemia is associated with an internal malignancy. 110) Multiple such lesions are associated with:: Image Link 1 Image Link 2 A. Cowden'sCorrect Choice B. Gorlin's C. Werner's D. Bloom's E. Sipple's Multiple sclerotic fibromas are seen in Cowden’s syndrome. 111) The diagnosis is: A. Dermatofibrosarcoma B. Angiolipoma Correct Choice C. Epithelioid sarcoma D. Nodular fasciitis E. Liposarcoma NEEDS EXPLANATIONS 112) Cellular neurothekeoma stains with: A. Low molecular weight keratin B. Stromelysin-3 C. Desmin D. PGP-9.5 Correct Choice E. S-100 PGP-9.5 and S100-a6 stains cellular neurothekeoma. Stromelysin-3 is positive in dermatofibromas and negative in dermatofibrosarcoma protuberans. Desmin stains rhabdomyosarcoma. S-100 stains neural tumors and melanocytic tumors among other things, but cellular neurothekeomas are generally S100-negative 30
  • 31. 113) What stain may be used to differentiate this entity from metastatic oat cell carcinoma of the lung? A. HMB 45 B. CEA C. PAS D. Cytokeratin 20Correct Choice E. S-100 Merkel cell carcinoma is a neuroendocrine cancer, usually of the head and neck. The tumor stain with cytokeratin 20 which is expressed in a paranuclear dot-like pattern. This stain helps to differentiate Merkel cell carcinoma from metastatic oat cell carinoma of the lung. 114) The diagnosis is: Image Link 1 Image Link 2 Image Link 3 A. Epithelioid sarcoma B. Angiosarcoma C. Epithelioid hemangioendothelioma D. Kaposi's sarcomaCorrect Choice E. Aneurismal dermatofibroma In nodular Kaposi’s sarcoma, one sees a proliferation of spindle cells, often arranged in nodules separated by fibrous bands. On higher power examination of the spindle cells, numerous extravasated erythrocytes can be seen between the cells. Often, hemosiderin-filled macrophages and plasma cells can be seen as well. The spindle cells are packed closely together and often will have intracytoplasmic pink inclusions (erythrophagolysosomes 115) The diagnosis is: A. Psoriasis B. Pityriasis rubra pilarisCorrect Choice C. Inflammatory linear verrucous epidermal nevus D. Prurigo nodularis E. Ichthyosis NEEDS EXPLANATION 116) What is the diagnosis? A. Dilated poreCorrect Choice B. Fibrofolliculoma C. Pilar sheath acanthoma D. Keratosis pilaris 31
  • 32. E. Trichoadenoma This is a dilated pore, in which there is an invagination lined by epidermis that is slightly acanthotic 117) These cells should stain with: Image Link 1 Image Link 2 Image Link 3 A. S100 B. factor XIIIa C. cytokeratin D. CD34 E. actinCorrect Choice This is a dermatomyofibroma. Dermatomyofibromas are often found over the scapula of women. The spindle cells are oriented parallel to the epidermis, and stain with vimentin and non-specific muscle actin. The spinde cells do not stain with desmin, S100, CD34, or Factor XIIIa 118) What is the diagnosis? A. Impetigo B. Bullous pemphigoidCorrect Choice C. Subcutaneous lupus erythematosus D. Dermatitis herpetiformis E. Polymorphous light eruption This is bullous pemphigoid, in which a subepidermal bullae/vesicle displays numerous eosinophils lining up at the dermo-epidermal junction 119) This developed in a patient with a history of breast cancer s/p surgical excision/radiation. Her diagnosis is: Image Link 1 Image Link 2 Image Link 3 A. Horner's B. Stewart-TrevesCorrect Choice C. Parkes-Weber D. Kettle's E. Klippel-Trenaunay Angiosarcoma can develop in a lymphedematous extremity. When it develops in the upper extremity after surgical treatment of breast cancer, it is referred to as Stewart-Treves syndrome. When it develops in the lower extremity after lymph node dissection for a melanoma, it is referred to as Kettle's syndrome 120) What is the diagnosis? A. Trichofolliculoma 32
  • 33. B. Spiradenoma C. Trichilemmoma D. Mixed tumor E. TrichoepithliomaCorrect Choice This is a trichoepithelioma, in which there are islands of basaloid cells in a somewhat fibrous stroma with no retraction between the islands and the stroma. Often horn cysts are seen (not shown). 121) The inclusions in infantile digital fibromatosis stain for trichrome and: A. Thioflavin T B. Pentahydroxy flavanol C. Osmium tetroxide D. Phosphotungstic acid hematoxylin Correct Choice E. Bodian Osmium tetroxide stains fat. Thioflavin T stains amyloid. The Bodian stain is for nerves. Pentahydroxy flavanol is a fluorescent stain for calcium 122) The diagnosis is: Image Link 1 Image Link 2 Image Link 3 Image Link 4 A. myofibromatosis B. schwannoma C. Kaposi's sarcoma D. neurofibromaCorrect Choice E. leiomyoma This is a plexiform neurofibroma. There are discrete nodules of spindle cells within the dermis. On higher power, the cells have wavy nuclei with pink cytoplasm 123) What is the diagnosis? A. Carbon tattooCorrect Choice B. Monsel’s reaction C. Blue nevus D. Postinflammatory hyperpigmentation E. Minocycline-induced hyperpigmentation This is a carbon tattoo, in which there are extracellular and intracellular particles of black material in the superficial dermis 124) What is the diagnosis? A. Deep penetrating nevus 33
  • 34. B. Recurrent nevus C. Congenital nevusCorrect Choice D. Epithelioid blue nevus E. Nevoid melanoma This is a congenital nevus, in which there are nests of nevomelanocytic cells at the dermoepidermal junction and extending deep into the dermis. In the deeper dermis, the cells infiltrate through collagen bundles and extend around adnexal structures. There is hyperkeratosis, acanthosis, and papillomatosis of the surface epidermis 125) The green color in chloroma is secondary to: A. Stromelysin B. Myeloperoxidase Correct Choice C. Fumarase D. Chloracetate E. Alkaline phosphatase Chloromas are greenish tumor grossly secondary to involvement of the skin in acute granulocytic leukemia. The green color is secondary to myeloperoxidase 126) Supporting evidence for the diagnosis of mycosis fungoides is CD4+ lymphocytes with loss of CD7 as well as loss of A. CD20 B. CD2 C. CD30 D. CD5 Correct Choice E. CD3 CD5 as well as CD7 are sometimes lost on the surface of epidermotropic T cells in mycosis fungoides. CD2, CD3, and CD5 are T cell markers. CD20 is a B cell marker. CD30 is positive in anaplastic large cell lymphoma cells, Hodgkin’s lymphoma, and lymphomatoid papulosis. Reactive infiltrates can also have some CD30-positive cells 127) Clinically, a nondescript hyperkeratotic papule on the ulnar side of the base of the fifth finger is most likely: A. Cutaneous horn B. Accessory digit Correct Choice C. Digital fibromatosis D. Acquired digital fibrokeratoma E. Glomus tumor Accessory digits (supernumerary digits) are usually found at the base of the fifth finger, often bilaterally 128) Multiple trichoepitheliomas are seen in all except: 34
  • 35. A. Brooke-Fordyce syndrome B. Rombo syndrome C. Gorlin's syndrome Correct Choice D. Brooke-Spiegler syndrome E. Bazex's syndrome Gorlin's syndrome is nevoid basal cell carcinoma syndrome; multiple trichoepitheliomas are not seen. Several syndromes have been associated with multiple trichoepitheliomas: Basex, Brooke- Fordyce, Brooke-Spiegler, Rombo, and possibly Rasmussen. (Rasmussen described one family in 1975 with autosomal dominant inheritance of multiple trichoepitheliomas, milia, and cylindromas.) Basex syndrome (follicular atrophoderma, hypotrichosis, occasional trichoepitheliomas, basal cell carcinomas, and localized or generalized hypohidrosis) is inherited in an X-linked dominant manner. Brooke and Fordyce both described multiple trichoepitheliomas concurrently in 1892, and therefore multiple familial trichoepitheliomas are sometimes called “Brooke-Fordyce” sydrome. Spiegler described patients with multiple cylindromas in 1899 and also noted that many of these patients had mutiple trichoepitheliomas; more recently it has been noted that multiple spiradenomas may be seen in patients with multiple trichoepitheliomas and cylinidromas; this co-occurrence of tumors has been referred to as “Brooke-Spiegler” syndrome. (Brooke-Fordyce and Brooke-Spiegler are likely the same syndrome.) Rombo syndrome is characterized by vermiculate atrophoderma, multiple BCCs, multiple trichoepitheliomas, cyanosis and peripheral vasodilation 129) The diagnosis is:: Image Link 1 Image Link 2 Image Link 3 Image Link 4 A. hypertrophic scarCorrect Choice B. leiomyoma C. dermatomyofibroma D. neurofibroma E. dermatofibroma 130) What is the diagnosis? A. Adenoid cystic carcinoma B. Trichoblastoma C. Microcystic adnexal carcinoma D. Trichodiscoma E. Morpheaform basal cell carcinomaCorrect Choice This is a morpheaform basal cell carcinoma, in which there are very infiltrative islands of basaloid cells, extending deep into the dermis. Around some of the basaloid islands, there is new, pink collagen. The basaloid proliferation off of the surface of the epidermis is more typical of a superficial multicentric basal cell carcinoma and aids in the diagnosis 131) This patient should be examined for:: Image Link 1 Image Link 2 Image Link 3 Image Link 4 35
  • 36. A. photosensitivity B. keratoacanthomasCorrect Choice C. pigmentary anomalies D. odontogenic cysts E. arsenical keratoses The figures show a sebaceous adenoma. Sebaceous adenomas are associated with Muir-Torre syndrome, in which patients can have an internal malignancy and multiple keratoacanthomas 132) The most likely diagnosis for this lesion would be: A. Verruca B. Acrochordon C. Acquired digital fibrokeratomaCorrect Choice D. Amputation neuroma E. Supernumery digit The diagnosis of this acral lesion is an acquired digital fibrokeratoma which shows a small, exophytic circumscribed lesions. Collagen bundles are oriented perpendicularly to the skin surface. The lesion lacks nerve twigs and bone, which may be present in supernumery digit or amputation neuroma 133) Clear cell syringomas are associated with: A. Sarcoidosis B. Malignancy C. Diabetes Correct Choice D. Lichen myxedematosis E. Argyria Clear cell syringomas are associated with diabetes. Syringomas are associated with Down’s syndrome 134) The diagnosis is: Image Link 1 Image Link 2 Image Link 3 A. tubular apocrine adenoma B. dermal duct tumor C. syringoma D. trichoepithelioma E. adenoid cystic carcinomaCorrect Choice In adenoid cystic carcinoma, there are cords and tubules of basaloid cells, arranged in a cribiform pattern. The cystic spaces contain a bluish material. Perineural invasion is common. The cells stain positively for epithelial membrane antigen (EMA). 135) What is the cause of this pigmentary condition? 36
  • 37. A. Hemochromatosis B. Post-inflammatory hyperpigmetation C. Minocycline ingestion D. ArgyriaCorrect Choice E. Chrysiasis The characteristic feature of argyria is the presence of black granules in the eccrine glands. It differentiates this condition from other pigmentary disorders 136) The diagnosis is: Image Link 1 Image Link 2 Image Link 3 Image Link 4 A. Erythropoietic protoporphyria B. Lipoid proteinosis C. Radiation dermatitis D. Colloid milium E. Lichen sclerosus et atrophicusCorrect Choice In lichen sclerosus et atrophicus, there is often an atrophic epidermis with overlying orthokeratosis that is thicker than the stratum spinosum, with some follicular plugging. There is sometimes a subepidermal separation. The upper dermis is homogenized and pink/pale. Sometimes underlying the homogenized zone, there is a band-like infiltrate (not seen here 137) A patient with a blue-red discoloration of the nail plate reports that the same finger becomes very tender when exposed to the cold. You suspect a: A. Pyogenic granuloma B. Glomus tumorCorrect Choice C. Periungual verruca D. Mucous cyst E. Pterygium Glomus tumors are tumors of the arterio-venous anastamosis of the digital dermis. They occur most frequently in the nail bed. The commonly have a bluish-red discoloration and may be tender or painful with exposure to heat or cold 138) Verruciform xanthoma is seen most commonly on: A. Distal extremities B. Mucosal surfaces and trunk/proximal extremities C. Oral mucosae and genital areas Correct Choice D. Nail bed and periungual areas E. Head and neck NEEDS EXPLANATION 37
  • 38. 139) This patient developed an acute vesicular rash after eating a mango. She has returned for a routine follow-up. She needs to be careful of exposure to: A. All of these answers are correct B. Ginkgo fruitCorrect Choice C. Croton D. Ragweed E. Tea tree oil Patients allergic to the peel of a mango can also be allergic to other plants/products of the Anacardiaceae family. Cross-reactions can occur with exposure to any plants of the genus Toxicodendron, to the oil from the cashew nut shell, to the Brazilian pepper tree, to lacquer from the Japanese lacquer tree, to ink from the Indian marking nut, and to the fruit pulp of the ginkgo tree, and others 140) The diagnosis is: Image Link 1 Image Link 2 Image Link 3 Image Link 4 A. Blastomycosis B. Cryptococcosis C. Histoplasmosis D. Toxoplasmosis E. LeishmaniaisisCorrect Choice The diagnosis is Leishmaniasis. In this condition, the epidermis is often ulcerated (not seen in this case) with a dense infiltrate is seen within the dermis. There are numerous macrophages (Leishman-Donovan bodies) within the infiltrate that have intracellular amastigotes within them. On close examination of the parasites, a kinetoplast is evident. In contrast to histoplasmosis, a discrete halo is not seen around the amastigotes 141) Caterpillar bodies are seen in: A. Dyskeratosis congenital B. Porphyria cutanea tarda Correct Choice C. Amyloidosis D. Lipoid proteinosis E. Mucocele Caterpillar bodies are thought to be type IV collagen 142) A healthy 6 month old girl has a subcutaneous nodule above her right eyebrow. A skin biopsy demonstrates a cystic lesion with adnexal structures in the wall. Your diagnosis is: A. Nevus sebaceous B. Epidermal inclusion cyst C. Steatocystoma 38
  • 39. D. Pilar cyst E. Dermoid cystCorrect Choice Dermoid cysts present along lines of embryonic closure. The are most commonly found on the head (around the eyes) and the neck. They are lined by an epidermis that contains various epidermal appendages that are usually fully matured 143) The diagnosis is: Image Link 1 Image Link 2 Image Link 3 A. basaloid squamous cell carcinoma B. malignant acrospiroma C. poroid squamous cell carcinoma D. porocarcinoma E. basosquamous carcinomaCorrect Choice In basosquamous carcinomas, there are areas that appear typical of basal cell carcinoma (with basaloid cells coming off the epidermis in buds with peripheral palisading) as well as areas typical of squamous cell carcinoma (with atypical keratinocytes that are more pink and angular than basaloid cells). Often, ulcerated basal cell carcinomas have squamous differentiation at the base of the ulcer and this is not to be confused with a basosquamous carcinoma, in which there are areas of both basal cell carcinoma and squamous cell carcinoma 144) The diagnosis is: A. Dermatofibroma Correct Choice B. Dermatomyofibroma C. Neurofibroma D. Plexiform fibrohistiocytic tumor E. Infantile digital fibromatosis NEEDS EXPLANATIONS 39